90
1 SỞ GIÁO DỤC VÀ ĐÀO TẠO NAM ĐỊNH TẬP SAN GIÁO DỤC SỐ 1 Môn: Toán (Lưu hành nội bộ) Ban biên soạn: Hội đồng chuyên môn tỉnh Nam Định Nam Định, tháng 5 năm 2019

SỞ GIÁO DỤC VÀ ĐÀO TẠO NAM ĐỊNH

  • Upload
    others

  • View
    2

  • Download
    0

Embed Size (px)

Citation preview

Page 1: SỞ GIÁO DỤC VÀ ĐÀO TẠO NAM ĐỊNH

1

SỞ GIÁO DỤC VÀ ĐÀO TẠO NAM ĐỊNH

TẬP SAN GIÁO DỤC SỐ 1

Môn: Toán

(Lưu hành nội bộ)

Ban biên soạn:

Hội đồng chuyên môn tỉnh Nam Định

Nam Định, tháng 5 năm 2019

Page 2: SỞ GIÁO DỤC VÀ ĐÀO TẠO NAM ĐỊNH

2

Biên soạn

1. Nguyễn Hoàng Cương – TTCM trường THPT chuyên Lê Hồng Phong

2. Nguyễn Trung Kiên – GV trường THPT chuyên Lê Hồng Phong

3. Nguyễn Trung Hiếu – PHT trường THPT A Hải Hậu

4. Lại Đức Thắng – TTCM trường THPT Giao Thủy

5. Nguyễn Hữu Thiêm – Phó trưởng phòng GDTrH Sở GDĐT

6. Nguyễn Quốc Tuấn – TTCM trường THPT Phạm Văn Nghị

7. Phạm Đình Hòe – chuyên viên phòng GDĐT Trực Ninh

8. Phạm Văn Thuận – GV trường THCS Nghĩa Hưng

9. Nguyễn Văn Tuyến – GV trường THCS Lê Quý Đôn

10. Phan Văn Vũ – GV trường THCS Trần Đăng Ninh

Biên tập

Nguyễn Hữu Thiêm – Phó trưởng phòng GDTrH Sở GDĐT

Nguyễn Hoàng Cương – TTCM trường THPT chuyên Lê Hồng Phong

Page 3: SỞ GIÁO DỤC VÀ ĐÀO TẠO NAM ĐỊNH

3

MỤC LỤC

Phần Nội dung Trang

PHẦN I.

MỘT SỐ CHUYÊN

ĐỂ ÔN TẬP

Chuyên đề 1. Căn thức bậc hai 5

Chuyên đề 2. Hàm số, phương trình bậc hai 11

Chuyên đề 3. Hệ phương trình 26

Chuyên đề 4. Phương trình chứa căn 30

Chuyên đề 5. Hình học phẳng 40

Chuyên đề 6. Bất đẳng thức 51

PHẦN II.

CÁC ĐỀ LUYỆN

THI VÀO LỚP 10

Đề số 01 57

Đề số 02 58

Đề số 03 59

Đề số 04 60

Đề số 05 61

Đề số 06 62

Đề số 07 63

Đề số 08 64

Đề số 09 65

Hướng dẫn - Đáp án 67

Page 4: SỞ GIÁO DỤC VÀ ĐÀO TẠO NAM ĐỊNH

4

LỜI GIỚI THIỆU

Căn cứ Quy chế về tổ chức và hoạt động của Hội đồng chuyên môn Giáo dục

Trung học ban hành kèm theo Quyết định số 481/QĐ-SGDĐT ngày 26 tháng 02 năm 2018 của Giám đốc Sở Giáo dục và Đào tạo (GDĐT); thực hiện Kế hoạch hoạt động Hội đồng chuyên môn Giáo dục trung học (GDTrH) giai đoạn 2018-2023 ban hành tại văn bản số 1487/SGDĐT-GDTrH ngày 30/10/2018, Hội đồng chuyên môn GDTrH của Sở GDĐT đã xây dựng tập san của 03 môn học Ngữ văn, Toán, tiếng Anh.

Đây là tập san đầu tay của Hội đồng chuyên môn GDTrH sau một năm triển khai hoạt động; là thành quả lao động miệt mài, tâm huyết của các thành viên trong Hội đồng chuyên môn.

Tập san môn Toán gồm 02 phần: Phần I. Các chuyên đề ôn thi vào lớp 10 kèm bài tập thực hành.

Phần II. Các đề luyện thi vào 10 (trường THPT chuyên Lê Hồng Phong và

trường THPT không chuyên).

Qua tập san, Hội đồng chuyên môn muốn chia sẻ với giáo viên trong toàn tỉnh (đặc biệt là giáo viên đang dạy các môn Ngữ văn, Toán, tiếng Anh) kinh nghiệm biên soạn chuyên đề ôn luyện cho học sinh thi vào lớp 10; kinh nghiệm biên soạn đề bài và đáp án theo định hướng đánh giá năng lực của học sinh. Tập san cũng là một tài liệu tham khảo bổ ích, thiết thực giúp các em học sinh đang học lớp 9 biết cách ôn tập, hệ thống kiến thức; biết cách tự học, tự rèn luyện kĩ năng làm bài thi theo định hướng yêu cầu nội dung của kì tuyển sinh vào lớp 10 THPT năm học 2019-2020.

Hội đồng chuyên môn trân trọng giới thiệu đến toàn thể cán bộ giáo viên và học sinh tỉnh Nam Định Tập san giáo dục số 1 môn Ngữ văn, môn Toán và môn Tiếng Anh (dưới dạng bản mềm pdf). Đây là tài liệu chuyên môn lưu hành nội bộ, trong quá trình sử dụng, Hội đồng chuyên môn rất mong nhận được những ý kiến đóng góp tích cực bằng văn bản (gửi về email: [email protected]) để Hội đồng chuyên môn tiếp tục hoàn thiện nâng cao chất lượng tập san.

HỘI ĐỒNG CHUYÊN MÔN

SỞ GIÁO DỤC VÀ ĐÀO TẠO NAM ĐỊNH

Page 5: SỞ GIÁO DỤC VÀ ĐÀO TẠO NAM ĐỊNH

5

PHẦN I - MỘT SỐ CHUYÊN ĐỀ ÔN TẬP

Chuyên đề 1

CĂN BẬC HAI VÀ MỘT SỐ VẤN ĐỀ LIÊN QUAN I. Kiến thức 1) Định nghĩa, tính chất của căn bậc hai

a) Căn bậc hai của số thực a không âm là số x sao cho 2 .x a

b) Với số dương a, số a được gọi là căn bậc hai số học của a.

c) Với 0a ta có 2

0xx a

x a

d) Với hai số a và b không âm, ta có a b a b . 2) ĐKXĐ của căn thức bậc hai:

A có nghĩa (hay xác định hay tồn tại) 0A

3) Các công thức biến đổi căn thức

1. 2A A

2. . .A B A B với , 0.A B

3. A A

B B với 0A và 0.B

4. 2.A B A B với 0.B

5. 2A B A B với , 0.A B

2A B A B với 0; 0.A B

6. 1A

ABB B

với 0, 0.AB B

7. 1A

ABB B

với 0, 0.AB B

II. Một số ví dụ Ví dụ 1. Tìm ĐKXĐ của các biểu thức sau:

a) 32 x b) 12

3

x c)

1

2

x d) 2 10 25.x x

Hướng dẫn:

a) ĐKXĐ của biểu thức là 3

2 3 0 .2

x x

b) ĐKXĐ của biểu thức là1

2 1 0 .2

x x

c) ĐKXĐ của biểu thức là 0 0

.11 0

x x

xx

d) ĐKXĐ của biểu thức là 22 10 25 0 5 0 5.x x x x

Ví dụ 2. Rút gọn biểu thức sau:

Page 6: SỞ GIÁO DỤC VÀ ĐÀO TẠO NAM ĐỊNH

6

2 2

) 2 2 5 2 5 ;

) 9 4 5 6 2 5;

) 2 3 2 3;

a A

b B

c C

2 2) 4 4 4 4d D a a a a với 2 2.a

Hướng dẫn:

2 2

2 2

) 2 2 5 2 5

2 2 5 2 5 2 2 5 5 2 2 2 2.

) 9 4 5 6 2 5

2 5 1 5 2 5 1 5 2 5 5 1

2 5 5 1 3.

a A

b B

c) Cách 1:

2 2

2 3 2 3

2 4 2 3 4 2 3 1 3 1 3 1 3 1 3

1 3 3 1 2 3

6.

C

C

C

Cách 2:

2

2

2 3 2 3

2 3 2 3 2 3 2 2 3 2 3 2 3 6

6

6

C

C

C

C

Vì 0 6.C C

2 22 2) 4 4 4 4 2 2 2 2 2 2d D a a a a a a a a a a

(vì 2 2a ). Do đó 4.D Ví dụ 3. Rút gọn biểu thức:

2 2)

3 1 3 1a A

;

2 2 1) : ;

5 3 5 3 3

2 3 2 3) .

2 3 2 3

b B

c C

Hướng dẫn:

Page 7: SỞ GIÁO DỤC VÀ ĐÀO TẠO NAM ĐỊNH

7

a)

2 3 1 2 3 12 2

3 1 3 1 2.3 1 3 1 3 1 3 1 3 1 3 1

A

b)

2 5 3 2 5 32 2 1

: . 3 5 3 5 3 . 35 3 5 35 3 5 3 3

B

2 3. 3 6.

c)

2 2

2 2

3 1 3 14 2 3 4 2 3 3 1 3 1

4 2 3 4 2 3 3 1 3 13 1 3 1C

2 2

3 1 3 14.

2 2

Ví dụ 4. Cho 3 2.x Tính giá trị của biểu thức 5 4 3 23 3 6 20 2024.B x x x x x Hướng dẫn:

Ta có 23 2 2 3 4 1 0.x x x x

5 4 3 2

5 4 3 4 3 2 2

3 2 2 2 2

2 3 2

3 3 6 20 2024

4 4 5 20 5 2019

4 1 4 1 5 4 1 2019

4 1 5 2019

B x x x x x

x x x x x x x x

x x x x x x x x

x x x x

Từ đó tính được 2019.B

Ví dụ 5. Cho biểu thức 1 7 1

:42 2

Axx x

với 0, 1.x x

a) Rút gọn biểu thức A.

b) Tính giá trị của biểu thức với 2 2

.2 3 2 3

x

Hướng dẫn: a) Với 0, 1x x ta có

1 7 1 1 7: . 2

42 2 2 2 2

2 7 5. 2 .

22 2

A xxx x x x x

x xx

xx x

b) Ta có

2 2

2 2 4 4 4 4

2 3 2 3 4 2 3 4 2 3 1 3 1 3x

2 23 1 3 1 2

3 1 3 1

thỏa mãn điều kiện.

Thay vào biểu thức 5

2

xA

x

ta được

2 5 8 3 2.

22 2A

Ví dụ 6. Cho biểu thức 2 2 1

:1 1 1

x xB

x xx x x x x

với 0, 1.x x

a) Rút gọn biểu thức B.

Page 8: SỞ GIÁO DỤC VÀ ĐÀO TẠO NAM ĐỊNH

8

b) Tìm x để 1

.2

B

Hướng dẫn: a) Với 0, 1x x ta có:

2 2 1:

1 1 1

2 2 1:

1 11 1 1

x xB

x xx x x x x

x x

xx xx x x x

1 2 1 2 12: . .

2 11 1 1 1 1

x x x x x x xx x x

x x xx x x x x x

b) Theo câu a) ta có 1

xB

x

với 0, 1x x .

1 12 1 0 1 2 1 0

2 21

xB x x x x

x

Từ đó tìm được 1

4x (thỏa mãn điều kiện).

Ví dụ 7. Cho biểu thức 2 4 8

3 4 1 4

x xB

x x x x

với 0, 16.x x

a) Rút gọn B. b) Tìm giá trị nguyên của x để biểu thức B nhận giá trị nguyên. c) Tìm x để giá trị của B là một số nguyên.

Hướng dẫn:

a) Rút gọn biểu thức B ta được 3

.1

xB

x

b) Ta có 3 3

31 1

xB

x x

với 0, 16.x x

Trường hợp 1: x không là số chính phương 1x là số vô tỉ B không nhận giá

trị nguyên.

Trường hợp 2: x là số chính phương 1x là số nguyên.

B nhận giá trị nguyên khi 3

1x nhận giá trị nguyên 1x là ước dương của 3

) 1 1 0x x (thỏa mãn điều kiện)

) 1 3 4x x (thỏa mãn điều kiện)

Kết luận: 0; 4.x x

c) Ta có 3 3

31 1

xB

x x

với 0, 16.x x Suy ra 3.B

Dễ chứng minh được 0.B Từ đó suy ra 0 3.B

Page 9: SỞ GIÁO DỤC VÀ ĐÀO TẠO NAM ĐỊNH

9

Mà B nhận giá rị nguyên 0;1;2B . Từ đó tìm được 1

0; ;44

x

.

III. Bài tập

1. Rút gọn các biểu thức sau

A = 6123321615

B = 5353

C = 21 6 6 21 6 6

D = 52104 52104

2. So sánh

a) 2020 2019A và 2018 2017;B

b) 2020 2018C và 2 2019.D

3. Cho hai biểu thức 2A x và 2 2

2 2

xB

x x x

với 0; 4x x

a) Tính giá trị của biểu thức A với 6 2 5.x

b) Rút gọn biểu thức B.

c) Tìm tất cả giá trị nguyên của x để .P A B nhận giá trị nguyên.

3. Tính giá trị của biểu thức: 2) 2 3a A x x y y với 5 2; 6 2 5.x y

)b B x y với ,x y thỏa mãn 2 22019 2019 2019.x x y y

3) 3 2015c C x x với 3 32 5 2 5x .

4. Cho biểu thức P = 1

)1(22

1

2

x

x

x

xx

xx

xx

a) Rút gọn P. b) Tìm giá trị nhỏ nhất của P.

c) Tìm x để biểu thức P

xQ

2 nhận giá trị là số nguyên.

5. Cho các biểu thức 2 3 3

93 3

x x xA

xx x

1

3

xB

x

với 0, 9.x x

a) Tính giá trị của biểu thức B tại 11 6 2.x b) Rút gọn biểu thức A.

c) Đặt .A

CB

Tìm giá trị của x để 2

.3

C

d) Đặt 4 7

. .3

xD x C

x

Tìm giá trị nhỏ nhất của D.

Chuyên đề 2. Giải bài toán bằng cách lập phương trình, lập hệ phương trình I. Tóm tắt lý thuyết

Các bước giải bài toán bằng cách lập phương trình, lập hệ phương trình: Bước 1. Lập phương trình (hệ phương trình) - Chọn ẩn và đặt điều kiện cho ẩn;

Page 10: SỞ GIÁO DỤC VÀ ĐÀO TẠO NAM ĐỊNH

10

- Biểu diễn các đại lượng chưa biết qua ẩn và các đại lượng đã biết; - Lập phương trình (hệ phương trình) biểu thị mối quan hệ giữa các đại lượng. Bước 2. Giải phương trình (hệ phương trình) vừa tìm được. Bước 3. Kết luận: Nhận định kết quả và trả lời. II. Ví dụ

Ví dụ 1. Một số có hai chữ số. Tỉ số giữa chữ số hàng chục và chữ số hàng đơn vị là 3

.4

Nếu cộng thêm 2 vào chữ số hàng chục thì được một chữ số bằng chữ số hàng đơn vị. Tìm số đã cho. Hướng dẫn: Gọi chữ số hàng chục và chữ số hàng đơn vị của số cần tìm lần lượt là a và b

, ,0 , 9a b a b .

Tỉ số giữa chữ số hàng chục và chữ số hàng đơn vị là 3

4 nên ta có phương trình (1):

3

4

a

b .

Nếu cộng thêm 2 vào chữ số hàng chục thì được một chữ số bằng chữ số hàng đơn vị nên ta có phương trình (2): 2 .a b

Giải hệ phương trình 3

4

2

a

b

a b

ta được 6

8

a

b

(thỏa mãn điều kiện của ẩn)

Vậy số cần tìm là 68. Ví dụ 2. Một đội thợ mỏ theo kế hoạch phải khai thác 350m than mỗi ngày. Nhưng khi

thực hiện mỗi ngày đội khai thác được 360m . Do đó đội đã hoàn thành trước kế hoạch 2

ngày và vượt mức 320m . Tính khối lượng than đội phải khai thác theo kế hoạch.

Hướng dẫn: Khối lượng than

khai thác (m3) Khối lượng than khai

thác 1 ngày (m3) Thời gian hoàn thành

công việc (ngày) Theo kế hoạch

x 350m 50

x

Thực tế 20x 360m 20

60

x

Giải phương trình 20

250 60

x x ta được 700x (thỏa mãn điều kiện của ẩn)

Kết luận: Khối lượng than đội phải khai thác theo kế hoạch là 3700 .x m

Ví dụ 3. Một ca nô xuôi dòng từ bến A đến bến B mất 4 giờ và ngược dòng từ bến B đến bến A mất 5 giờ. Tính khoảng cách giữa hai bến, biết vận tốc của dòng nước là 2 / .km h

Hướng dẫn: Quãng đường ( )km Vận tốc /km h Thời gian ( )h

Xuôi dòng x

4

x

4

Ngược dòng x

5

x

5

Giải phương trình 44 5

x x ta được 80x (thỏa mãn điều kiện của ẩn)

Kết luận: Khoảng cách giữa hai bến là 80 .km

Page 11: SỞ GIÁO DỤC VÀ ĐÀO TẠO NAM ĐỊNH

11

Ví dụ 4. Hai đội công nhân cùng xây một ngôi nhà thì sẽ hoàn thành trong 24 ngày. Mỗi

ngày phần việc làm được của đội I bằng 3

2 phần việc đội II làm được. Hỏi nếu làm một

mình thì mỗi đội sẽ xây xong ngôi nhà trong bao nhiêu ngày? Hướng dẫn:

Đội I Đội II Cả hai đội Số ngày x y 24

Phần việc làm trong 1 ngày

1

x

1

y

1

24

Giải hệ phương trình

1 3 1.

2

1 1 1

24

x y

x y

ta được 40

60

x

y

(thỏa mãn điều kiện của ẩn)

Kết luận: Đội I xây xong ngôi nhà trong 40 ngày và đội II xây xong ngôi nhà trong 60 ngày. III. Bài tập:

1. Một người dự định đi từ Hà Nội về Nam Định. Ban đầu người đó dự định đi xe máy với vận tốc 50km/h. Nhưng sau đó người đó lại đi ô tô với vận tốc 60km/h nên đã đến sớm hơn dự định là 30 phút. Tính quãng đường từ Hà Nội - Nam Định. 2. Một ca nô chạy xuôi dòng từ A đến B hết 1 giờ 30 phút và ngược dòng từ B đến A hết 2 giờ. Tính vận tốc của ca nô khi nước yên lặng, biết rằng quãng sông AB dài 50 km và vận tốc dòng nước là 2 km/h. 3. Một đội máy kéo dự định mỗi ngày cày được 40 ha. Khi thực hiện mỗi ngày đội cày được 52 ha. Vì vậy đội không những cày xong trước thời hạn 2 ngày mà còn cày thêm được 4 ha nữa. Tính diện tích ruộng mà đội phải cày theo kế hoạch. 4. Theo kế hoạch hai tổ sản xuất phải làm được 900 chi tiết máy trong một thời gian qui định. Do cải tiến kĩ thuật nên tổ I vượt mức 20%, tổ II vượt mức 10% so với kế hoạch. Vì vậy hai tổ sản xuất được 1010 chi tiết máy. Hỏi theo kế hoạch mỗi tổ sản xuất pahir làm bai nhiêu chi tiết máy? 5. Hiệu của hai số bằng 12. Nếu chia số bé cho 7 và số lớn cho 5 thì thương thứ nhất bé hơn thương thứ hai là 4. Tìm hai số. 6. Một ô tô du lịch đi từ A đến C. Cùng một lúc từ địa điểm B nằm trên đoạn đường AC, có một ô tô vận tải cùng đi đến C. Sau 5 giờ hai ô tô gặp nhau tại C. Hỏi ô tô du lịch đi từ A đến B mất bao lâu, biết rằng vận tốc của ô tô vận tải bằng 0,6 vận tốc của ô tô du lịch. 7. Cho một lượng dung dịch chứa 10% muối. Nếu pha thêm 200 g nước thì được một dung dịch 6%. Hỏi có bao nhiêu gam muối trong dung dịch đã cho?

Chuyên đề 2

HÀM SỐ BẬC NHẤT, BẬC HAI - PHƯƠNG TRÌNH BẬC HAI

A. TÓM TẮT LÝ THUYẾT

1. Khái niệm hàm số

Nếu đại lượng y phụ thuộc vào đại lượng thay đổi x sao cho với mỗi giá trị của x ta

luôn xác định được một và chỉ một giá trị tương ứng của y thì y được gọi là hàm số của x

và x được gọi là biến số .

Kí hiệu là , y f x y g x ,…

Page 12: SỞ GIÁO DỤC VÀ ĐÀO TẠO NAM ĐỊNH

12

Khi x thay đổi mà y luôn nhận một giá trị không đổi thì hàm số y được gọi là hàm

hằng.

2. Đồ thị của hàm số Trong mặt phẳng tọa độ Oxy, tập hợp tất cả các điểm biểu diễn các cặp giá trị tương

ứng ; x f x được gọi là đồ thị hàm số y = f x .

3. Tập xác định của hàm số

TXĐ của hàm số y f x là tập hợp các giá trị của biến để biểu thức f x có

nghĩa.

4. Hàm số đồng biến, hàm số nghịch biến

Cho hàm số y f x xác định với mọi x thuộc .

a) Nếu giá trị của biến x tăng lên mà giá trị tương ứng của f x cũng tăng theo thì

ta nói hàm số y f x là hàm số đồng biến trên . (hay với 1 2, x x bất kỳ thuộc ; nếu

1 2x x mà 1 2f x f x thì hàm số y f x đồng biến trên )

b) Nếu giá trị của biến x tăng lên mà giá trị tương ứng của f x lại giảm đi thì ta

nói hàm số y f x là hàm số nghịch biến trên . (hay với 1 2, x x bất kỳ thuộc ; nếu

1 2x x mà 1 2f x f x thì hàm số y f x nghịch biến trên )

5. Hàm số bậc nhất , 0y ax b a

a) Định nghĩa: Hàm số bậc nhất là hàm số được cho bởi công thức y ax b , trong

đó ,a b là các số cho trước , 0a .

Hàm số bậc nhất xác định với mọi x thuộc

b)Tính chất hàm số bậc nhất: Hàm số đồng biến trên khi 0a , nghịch biến trên

khi 0a .

Chú ý: Khi 0a , ta có hàm số y b là hàm hằng.

c) Đồ thị hàm số bậc nhất: Đồ thị hàm số bậc nhất , 0y ax b a là một đường

thẳng. Ta cũng gọi đồ thị của hàm số y ax b là đường thẳng y ax b . Đường thẳng này

có các đặc điểm sau

+ Cắt trục tung tại điểm 0; ,b b gọi là tung độ gốc của đường thẳng.

+ Cắt trục hoành tại điểm ;0b

a

.

Chú ý : Khi 0b , đồ thị đi qua gốc tọa độ.

Nếu 0a thì đường thẳng “đi lên” từ trái qua phải, nếu 0a thì đường thẳng “đi

xuống” từ trái qua phải.

d) Vị trí tương đối của hai đường thẳng

Cho hai đường thẳng , 0y ax b a và đường thẳng ' ', ' 0y a x b a

+ Hai đường thẳng song song với nhau khi và chỉ khi 'a a và 'b b

+ Hai đường thẳng trùng nhau khi và chỉ khi 'a a và 'b b

+ Hai đường thẳng cắt nhau khi và chỉ khi 'a a

Trường hợp riêng : Hai đường thẳng vuông góc với nhau khi và chỉ khi . ' 1a a

e) Hệ số góc của đường thẳng

Page 13: SỞ GIÁO DỤC VÀ ĐÀO TẠO NAM ĐỊNH

13

Trong mặt phẳng tọa độ Oxy cho đường thẳng , 0y ax b a . Khi ta nói góc

là góc tạo bởi đường thẳng y ax b và chiều dương trục Ox .

Ta gọi a là hệ số góc của đường thẳng y ax b

+ Nếu 0a thì là góc nhọn và a càng lớn thì góc càng lớn.

+ Nếu 0a thì là góc tù và a càng lớn thì góc càng lớn.

+ Nếu 0a thì tan a . Nếu 0a thì tan a .

6. Hàm số 2 ( 0) y ax a

a) Hàm số 2 y ax xác định với mọi x thuộc và có tính chất sau

+ Nếu 0a thì hàm số đồng biến khi 0 x và nghịch biến khi 0x .

+ Nếu 0a thì hàm số đồng biến khi 0x và nghịch biến khi 0 x .

b) Lưu ý về giá trị của hàm số

+ Nếu 0a thì ta có 2 0y ax với mọi x ( 0 y khi 0y ), nên giá trị nhỏ

nhất của hàm số là 0 y đạt được khi 0 x .

+ Nếu 0a thì ta có 2 0y ax với mọi x ( 0 y khi 0y ), nên giá trị lớn

nhất của hàm số là 0 y đạt được khi 0 x

c) Đồ thị của hàm số 2 0y ax a

Đồ thị hàm số 2 0y ax a là một đường parabol đỉnh O, nhận trục Oy làm trục

đối xứng.

+ Nếu 0a thì đồ thị nằm phía trên trục hoành và nhận O là điểm thấp nhất của đồ

thị.

+ Nếu 0a thì đồ thị nằm phía dưới trục hoành và nhận O là điểm cao nhất của đồ

thị.

7. Một số đường thẳng có phương trình đặc biệt

a) Đường thẳng có dạng y m

+ Nếu 0m thì y m là phương trình của đường thẳng song song với trục hoành.

+ Nếu 0m thì 0y là phương trình của trục hoành.

b) Đường thẳng có dạng x n

+ Nếu 0n thì x n là phương trình của đường thẳng song song với trục tung.

+ Nếu 0 n thì 0x là phương trình của trục tung.

B. BÀI TẬP

1) Tính đơn điệu của hàm số a) Kiến thức cần áp dụng : Tính đồng biến, nghịch biến của từng loại hàm số.

b) Ví dụ

Ví dụ 1: Tìm m để hàm số - 2018 2019y m x nghịch biến trên ?

Hướng dẫn

Hàm số - 2018 2019y m x nghịch biến trên - 2018 0 2018m m

Ví dụ 2: Trong các hàm số sau đây, hàm số nào đồng biến khi x dương và nghịch biến

khi x âm?

A. 21 3y x B. 2018 2019.y x C. 22 3 5y x D.

211

3y x

Page 14: SỞ GIÁO DỤC VÀ ĐÀO TẠO NAM ĐỊNH

14

Ví dụ 3: Trong các hàm số sau đây, hàm số nào đồng biến khi x < 0

A. 2( 2 1)y x B 24y x . C. 2(2 5)y x D.

2 1y x

(chú ý : Hàm số bậc nhất chỉ luôn đồng biến hoặc luôn nghịch biến

Do đó ta chỉ cần xét xem hàm số bậc hai nào có hệ số a dương)

Ví dụ 4: Cho 23 6 f x a x và 22 – 3g x a x . Tìm a để khi x < 0 thì hàm

số y f x đồng biến và hàm số y g x nghịch biến.

Hướng dẫn Yêu cầu bài toán3 6 0 3

22 – 3 0 2

aa

a

2) Vẽ đồ thị của hàm số

2.1. Vẽ đồ thị của hàm số ,y = ax+b (a 0)

a) Phương pháp

+ Nếu 0,b khi đó ta có y = ax

Xác định một điểm thuộc đồ thị của hàm số mà khác với gốc tọa độ, chẳng hạn điểm

1; .A a Vẽ đường thẳng OA ta được đồ thị của hàm số.

+ Nếu 0b

- Xác định hai điểm phân biệt thuộc đồ thị của hàm số.

Xác định điểm 0;A b là giao điểm với trục tung.

Xác định điểm ( ;0)b

Ba

là giao điểm với trục hoành.

-Vẽ đường thẳng đi qua hai điểm A và B ta được đồ thị của hàm số.

b) Ví dụ: Vẽ đồ thị của hàm số 2y = x+

- Cho 2x = 0 y = ; ta được điểm 0;2A là giao điểm của đồ thị hàm số với trục

tung.

Cho y 0 x 2; ta được điểm 2;0B là giao điểm của đồ thị hàm số với

trục hoành.

- Vẽ đường thẳng đi qua hai điểm 0;2A và 2;0B ta được đồ thị của hàm số

2y = x+

2.2. Vẽ đồ thị của hàm số 2 ( 0) y ax a

a) Phương pháp - Lập bảng một số cặp giá trị tương ứng giữa x và y (thường là 5 hoặc 7 cặp

giá trị ; trong đó x lấy giá trị 0 và các giá trị là các số đối nhau gần 0), chẳng hạn

x -2 -1 0 1 2

y = ax2 4a A 0 a 4a

- Biểu diễn các cặp giá trị tương ứng giữa x và y trong bảng trên mặt phẳng

tọa độ

- Vẽ đường cong đi qua các điểm đó ta được đồ thị của hàm số đó cho.

b) Ví dụ : Vẽ đồ thị của hàm số 22y x

- Bảng một số cặp giá trị tương ứng giữa x và y

Page 15: SỞ GIÁO DỤC VÀ ĐÀO TẠO NAM ĐỊNH

15

f(x)=2x^2

Series 1

-5 -4 -3 -2 -1 1 2 3 4 5 6

-2

-1

1

2

3

4

5

6

7

8

9y

- Đồ thị của hàm số đó cho là một parabol đi qua các điểm

(-2; 8); (-1; 2); (0; 0); (1; 2) và (2; 8).

3) Viết phương trình của đường thẳng (d) thỏa mãn điều kiện cho trước

a) Biết d song song với đường thẳng ' : , 0d y ax b a và đi qua điểm

0 0;A x y

Phương pháp

- Vì đường thẳng (d) song song với đường thẳng (d’) nên phương trình của đường

thẳng (d) có dạng : ', 'd y ax b b b

- Vì đường thẳng (d) đi qua điểm 0 0;A x y nên ta có : 0 0 'y ax b . Từ đó suy ra 'b ,

ta so sánh với điều kiện ’ .b b

- Kết luận về phương trình của đường thẳng (d).

Ví dụ 1: Viết phương trình của đường thẳng (d) song song với đường thẳng y = 2x + 1

và đi qua điểm A(1; 2).

Ví dụ 2: Viết phương trình đường thẳng (d) đi qua 1;1B có hệ số góc 2k .

b) Biết (d) đi qua hai điểm 1 1 2 2; , ;A x y B x y

Phương pháp

- Phương trình của đường thẳng (d) có dạng y ax b

- Vì d đi qua điểm 1 1;A x y nên ta có : 1 1y ax b

Vì d đi qua điểm 2 2;B x y nên ta có : 2 2y ax b

Do đó ta có hệ phương trình 1 1

2 2

y ax b

y ax b

- Giải hệ phương trình trên ta tìm được a và b , sau đó kết luận về phương trình của

đường thẳng d .

Ví dụ : Xác định hàm số y ax b , biết rằng đồ thị của hàm số đi qua hai điểm

1; 3 , 1; 1 .A B

Hướng dẫn

- Vì đồ thị của hàm số y ax b đi qua điểm 1; 3A nên ta có :

3 .1 3 1a b a b

Đồ thị của hàm số y ax b đi qua điểm 1; 1B nên ta có :

1 1 1a b a b

Từ (1) và (2) ta có hệ phương trình : 3

1

a b

a b

x -2 -1 0 1 2

y = 2x2 8 2 0 2 8

Page 16: SỞ GIÁO DỤC VÀ ĐÀO TẠO NAM ĐỊNH

16

- Giải hệ phương trình trên ta được 2; 1a b

Vậy hàm số cần tìm là 2 1y x

(Lưu ý : Nếu biết đồ thị của hàm số cắt trục tung tại điểm có tung độ bằng 0y thì có nghĩa

là đồ thị hàm số đi qua điểm 00; .y Đồ thị của hàm số cắt trục hoành tại điểm có hoành độ

bằng 0x có nghĩa là đồ thị của hàm số đi qua điểm 0;0x )

4) Xét vị trí tương đối của hai đường thẳng a) Phương pháp: Dựa vào điều kiện để hai đường thẳng song song, cắt nhau, trùng

nhau để làm.

Lưu ý : - Muốn tìm tọa độ giao điểm của hai đường thẳng, ta giải hệ phương trình gồm

hai phương trình của hai đường thẳng đó.

- Muốn tìm điều kiện để ba đường thẳng đồng quy, trước hết ta tìm tọa độ giao

điểm của hai đường thẳng đó có phương trình cụ thể, sau đó ta tìm điều kiện để đường

thẳng còn lại cũng đi qua giao điểm của hai đường thẳng đó.

b) Ví dụ: Xác định m để hai đường thẳng 2 2 3 y m x m và

2 2 2 1 y m x m song song với nhau.

Hướng dẫn

Điều kiện để hai đường thẳng đó cho song song với nhau là : 2 22 2 2 2 0

3 2 1 2

m m m m

m m m

0

02

2

m

mm

m

Vậy với 0m thì hai đường thẳng đó cho song song với nhau.

5) Xét vị trí tương đối của đường thẳng và parabol

a) Phương pháp

Tọa độ giao điểm của parabol 2 ( 0) y ax a và đường thẳng y mx n là nghiệm

của hệ phương trình

(I) 22 2 0 *

y mx ny mx n y mx n

ax mx ny ax ax mx n

+ Nếu phương trình (*) có hai nghiệm phân biệt thì hệ (I) có hai nghiệm phân biệt. Khi

đó đường thẳng cắt parabol tại hai điểm phân biệt, hai nghiệm của hệ (I) chính là tọa độ của

hai giao điểm.

+ Nếu phương trình (*) có nghiệm kép thì hệ (I) có nghiệm duy nhất. Khi đó đường

thẳng và parabol tiếp xúc với nhau, nghiệm duy nhất của hệ (I) chính là tọa độ của tiếp điểm

+ Nếu phương trình (*) vô nghiệm thì hệ (I) vô nghiệm. Khi đó đường thẳng và parabol

không có điểm chung.

b) Ví dụ

Ví dụ 1. Tìm tọa độ giao điểm của parabol 22y x P và đường thẳng 2 4y x D .

Hướng dẫn

Page 17: SỞ GIÁO DỤC VÀ ĐÀO TẠO NAM ĐỊNH

17

Hoành độ giao điểm của parabol (P) và đường thẳng (D) là nghiệm của phương trình

:

22 2 4.x x Giải phương trình này ta được hai ngiệm 1 21; 2x x

Vậy tọa độ các hai giao điểm của (P) và (D) là 1;2 và 2;8 .

Ví dụ 2. Với giá trị nào của a thì đường thẳng :d y x a tiếp xúc với parabol

2:P y x

A. 1

.4

a B. 1

.4

a C. 1

.4

a D. 1

.4

a

Ví dụ 3. Viết phương trình đường thẳng đi qua 1;2M và tiếp xúc với parabol 22y x

.

Hướng dẫn

Phương trình đường thẳng cần lập có dạng: y ax b d .

1;2 2 1 M d a b

d tiếp xúc với đường cong 22y x khi và chỉ khi phương trình 22x ax b có

nghiệm kép 2 8 0 2a b

Từ (1) và (2) ta có 4

2

a

b

. Vậy phương trình đường thẳng là 4 2y x

BÀI TẬP TỰ LUYỆN

Bài 1. Cho parabol 2: 2P y x và đường thẳng : 1d y x m ( với m là tham số).

1) Vẽ parabol P

2) Tìm tất cả các giá trị của m để P cắt d có đúng một điểm chung.

3) Tìm tọa độ các điểm thuộc P có hoành độ bằng hai lần tung độ.

Bài 2. Cho hàm số 2y x có đồ thị P và hàm số 4y x m có đồ thị md

1) Vẽ đồ thị P

2) Tìm tất cả các giá trị của m sao cho md và P cắt nhau tại hai điểm phân biệt,

trong đó tung độ của một trong hai giao điểm đó bằng 1.

Bài 3. Tìm a và b để đường thẳng : 2d y a x b có hệ số góc bằng 4 và đi qua

điểm 1;M

Bài 4. Cho Parabol P 2y x và đường thẳng (d): ( 1) 4y m x m (tham số m )

1) Với 2m , tìm tọa độ giao điểm của P và d

2) Tìm m để d cắt P tại hai điểm nằm về hai phía của trục tung.

Bài 5. Trong hệ toạ độ Oxy , gọi P là đồ thị của hàm số 2 y x và d là đồ thị của hàm

số 2y x

1) Vẽ các đồ thị P và d . Từ đó , xác định toạ độ giao điểm của P và d bằng đồ

thị .

Page 18: SỞ GIÁO DỤC VÀ ĐÀO TẠO NAM ĐỊNH

18

2) Tìm a và b để đồ thị của hàm số y ax b song song với d và cắt P tại

điểm có hoành độ bằng 1.

Bài 6. Cho parapol 2:P y x và đường thẳng 2: 2 1d y x m ( m là tham số).

1/ Xác định tất cả các giá trị của m để d song song với đường thẳng

2 2' : 2d y m x m m .

2/ Chứng minh rằng với mọi m , d luôn cắt P tại hai điểm phân biệt A và B.

3/ Ký hiệu ;A Bx x là hoành độ của điểm A và điểm B. Tìm m sao cho 2 2 14A Bx x .

Bài 7. Trong mặt phẳng, với hệ tọa độ Oxy, cho đường thẳng d có phương trình:

( 1)y m x n .

1) Với giá trị nào của m và n thì d song song với trục Ox .

2) Xác định phương trình của d , biết d đi qua điểm 1; 1A và có hệ số góc bằng

3.

Bài 8. Cho hai đường thẳng : 2 d y x m và 2' : 2 1d y m x

1) Khi 2,m tìm toạ độ giao điểm của chúng.

2) Tìm m để d song song với 'd

Bài 9. Tìm m để đường thẳng 3 6y x và đường thẳng 5

2 12

y x m cắt nhau tại

một điểm nằm trên trục hoành.

Bài 10. Cho hàm số 2 1 2y m x m

1) Tìm m để hàm số nghịch biến trên .

2) Tìm m để đồ thị hàm số đi qua 1;2A

Bài 11. Trong mặt phẳng toạ độ Oxy , với giá trị nào của ,a b thì đường thẳng

: 2d y ax b và đường thẳng ' : 3d y a x b song song với nhau.

Bài 12. Tìm tất cả các giá trị của m để 3 đường thẳng 3 2 4; 2x y x y m và 2 3x y

đồng quy

Bài 13. Biết đồ thị của hàm số 2(2 1)y m x đi qua điểm 2; 1M

1) Xác định giá trị của m . Vẽ đồ thị hàm số trên với giá trị m vừa tìm được?

2) Xác định tính đồng biến và nghịch biến của hàm số khi đó?

Bài 14. Cho hàm số 2.y ax

1) Xác định a , biết rằng đồ thị hàm số 2y ax cắt đường thẳng 4y x tại điểm A có

hoành độ là 4.

2) Với a tìm được, hãy vẽ đồ thị hàm số 2y ax và 4y x trên cùng một mặt

phẳng tọa độ.

Bài 15. Cho parabol P 2y x và đường thẳng (d) có hàm số 22( 1) 3y m x m

1) Khi 2m . Vẽ đồ thị hai hàm số trên cùng một hệ trục tọa độ.

2) Tìm m để đường thẳng d tiếp xúc với parabol P .

Page 19: SỞ GIÁO DỤC VÀ ĐÀO TẠO NAM ĐỊNH

19

Bài 16. Trong mặt phẳng tọa độ Oxy cho parabol P có phương trình 2y x và đường

thẳng d có phương trình 2y x m

1) Tìm tọa độ giao điểm của đường thẳng d và parabol P khi 3m

2) Tìm giá trị của m để d cắt P tại hai điểm phân biệt có hoành độ 1 2; x x thỏa mãn

hệ thức 2 21 2 6x x

Bài 17. Cho parabol P : 2y x và đường thẳng : 4d y mx ( m là tham số)

1) Chứng tỏ rằng: với mọi m đường thẳng d luôn cắt P tại hai điểm nằm về hai phía của

trục tung

2) Gọi 1 1;x y và 2 2;x y là tọa độ giao điểm của P và d , tìm m để:

1 2 2 11 1 6x y x y

Bài 18. Trong mặt phẳng tọa độ Oxy cho parabol P : 2y x và đường thẳng (d) :

3 1y x m ( m là tham số)

1) Tìm m để P và d tiếp xúc với nhau. Tìm tọa độ tiếp điểm của chúng.

2) Khi (d) và P cắt nhau tại hai điểm phân biệt, gọi hoành độ giao điểm của d và

P là Ax và xB . Tìm m để 2A Bx x

Bài 19. Trong mặt phẳng tọa độ Oxy cho parabol 2y x P và đường thẳng 2 4y x d

1) Chứng minh d và P luôn cắt nhau tại hai điểm phân biệt

2) Gọi 1 2,x x là hoành độ giao điểm của d và P . Tính giá trị của biểu thức

1 2 1 2A x x x x

Bài 20. Trong mặt phẳng tọa độ Oxy cho parabol P 22y x và một điểm M nằm trên P .

Biết rằng điểm M có tung độ bằng 8 và ở phía bên trái trục Oy . Hãy tìm tọa độ điểm M và

viết phương trình đường thẳng OM.

PHƯƠNG TRÌNH BẬC HAI

A. Tóm tắt lý thuyết

I. Định nghĩa: Phương trình bậc hai một ẩn là phương trình có dạng 2 0ax bx c trong

đó x là ẩn a,b,c là những số cho trước gọi là các hệ số và 0a

II. Công thức nghiệm của phương trình bậc hai

Phương trình bậc hai 2 0( 0)ax bx c a , 2 4b ac

+ Nếu 0 phương trình có hai nghiệm phân biệt : 1 2;2 2

b bx x

a a

+ Nếu 0 phương trình có nghiệm kép : 1 22

bx x

a

+ Nếu 0 phương trình vô nghiệm.

III. Công thức nghiệm thu gọn : Phương trình bậc hai 2 0( 0)ax bx c a và 2 'b b , 2' 'b ac

+ Nếu ' 0 phương trình có hai nghiệm phân biệt :

Page 20: SỞ GIÁO DỤC VÀ ĐÀO TẠO NAM ĐỊNH

20

1 2

' ' ' ';

b bx x

a a

+ Nếu ' 0 phương trình có nghiệm kép :

1 2

'bx x

a

+ Nếu ' 0 phương trình vô nghiệm.

IV. Hệ thức Viet và ứng dụng

1. Nếu 1 2;x x là hai nghiệm của phương trình 2 0( 0)ax bx c a thì :

1 2

1 2

bx x

a

cx x

a

2. Muốn tìm hai số u và v , biết ;u v S uv P , ta giải phương trình

2 0x Sx P (Điều kiện để có u và v là 2 4 0S P )

3. Nếu 0a b c thì phương trình 2 0( 0)ax bx c a có hai nghiệm : 1 21;c

x xa

Nếu 0a b c thì phương trình 2 0( 0)ax bx c a có hai nghiệm :

1 21;c

x xa

V. Một số quy tắc, phép biến đổi - Quy tắc nhân, chia đa thức.

- Hằng đẳng thức đáng nhớ.

- Các phương pháp phân tích đa thức thành nhân tử.

- Phương pháp quy đồng mẫu thức của hai hay nhiều phân thức. Các phép tính cộng, trừ,

nhân, chia các phân thức đại số.

- Quy tắc biến đổi phương trình, bất phương trình.

- Khái niệm căn bậc hai và các phép biến đổi đơn giản biểu thức chứa căn bậc hai.

- Phương pháp giải hệ phương trình.

B. BÀI TẬP

I. Phương trình bậc hai không có tham số

1. Phương trình bậc hai dạng khuyết

a) Phương trình bậc hai khuyết hạng tử bậc nhất Phương pháp giải :

- Chuyển hạng tử tự do sang vế phải.

- Chia cả hai vế cho hệ số bậc hai đưa về dạng: 2x a

+) a > 0 phương trình có nghiệm x a

+) a = 0 phương trình có nghiệm x = 0

+) a < 0 phương trình vô nghiệm

Ví dụ: Giải phương trình: 22 8 0x 2 2 22 8 0 2 8 4 2x x x x

Vậy phương trình có nghiệm 2x

b) Phương trình bậc hai khuyết hạng tử tự do

Page 21: SỞ GIÁO DỤC VÀ ĐÀO TẠO NAM ĐỊNH

21

Phương pháp giải: Phân tích đa thức vế trái thành nhân tử bằng phương pháp đặt nhân

tử chung, đưa về phương trình tích rồi giải.

Ví dụ: Giải phương trình: 23 5 0x x

2

00

3 5 0 (3 5) 53 5 0

3

xx

x x x xx x

Vậy phương trình có nghiệm 5

0;3

x x

2. Phương trình bậc hai đầy đủ Phương pháp giải :

- Sử dụng công thức nghiệm hoặc công thức nghiệm thu gọn để giải.

- Sử dụng quy tắc nhẩm nghiệm để tính nghiệm với một số phương trình đặc biệt.

Ví dụ: Giải phương trình: 22 3 5 0x x

Cách 1 : Sử dụng công thức nghiệm

23 4.( 2).5 9 40 49 0; 9

phương trình có hai nghiệm phân biệt

1 2

3 7 3 7 51;

2.( 2) 2.( 2) 2x x

Cách 2 : Nhẩm nghiệm

Ta có 2 3 5 0a b c phương trình có nghiệm : 1 2

5 51;

2 2x x

3. Phương trình đưa được về phương trình bậc hai

a) Phương trình trùng phương : 4 2 0( 0)ax bx c a

Phương pháp giải : Đặt 2t x ( 0t ) đưa về dạng : 2 0at bt c

Ví dụ: Giải phương trình: 4 23 4 0x x

Đặt 2 ( 0)t x t . Ta có phương trình : 2 3 4 0t t

Ta có 1 3 4 0 a b c

=> phương trình có nghiệm : 1 1 0t (thỏa mãn); 2

44 0

1t (loại)

21 1 1t x x

Vậy phương trình có nghiệm 1x

b) Phương trình chứa ẩn ở mẫu Phương pháp giải :

- Bước 1. Tìm điều kiện xác định của phương trình.

- Bước 2. Quy đồng mẫu thức hai vế rồi khử mẫu.

- Bước 3. Giải phương trình vừa nhận được.

- Bước 4. Trong các giá trị tìm được của ẩn, loại các giá trị không thỏa mãn điều kiện

xác định, các giá trị thỏa mãn điều kiện xác định là nghiệm của phương trình đã cho.

Ví dụ: Giải phương trình: 2 6

35 2

x

x x

(ĐKXĐ : 2; 5x x )

ta có 2 6

35 2

x

x x

( 2)(2 ) 3( 5)(2 ) 6( 5)

( 5)(2 ) ( 5)(2 ) ( 5)(2 )

x x x x x

x x x x x x

Page 22: SỞ GIÁO DỤC VÀ ĐÀO TẠO NAM ĐỊNH

22

2 2 2

2

( 2)(2 ) 3( 5)(2 ) 6( 5)( 2)(2 ) 3( 5)(2 ) 6( 5)

( 5)(2 ) ( 5)(2 ) ( 5)(2 )

4 6 3 30 15 6 30 4 15 4 0

15 4.( 4).4 225 64 289 0; 17

x x x x xx x x x x

x x x x x x

x x x x x x x

Phương trình có hai nghiệm 1

15 17 1

2.( 4) 4x

(thỏa mãn ĐKXĐ);

2

15 174

2.( 4)x

(thỏa mãn ĐKXĐ)

c) Phương trình tích Phương pháp giải: Ta phân tích phương trình bậc cao thành phương trình tích bậc 1;2

Ví dụ: Giải phương trình 3 2x 3x 2x 6 0 3 2 3 2 2 2

2 2

3 2 6 0 ( 3 ) (2 6) 0 ( 3) 2( 3) 0 ( 3)( 2) 0

33 0 3

2 0 2 2

x x x x x x x x x x x

xx x

x x x

Vậy phương trình có nghiệm 3; 2x x

II. Phương trình bậc hai có tham số

1. Giải phương trình khi biết giá trị của tham số 2. Tìm tham số biết số nghiệm của phương trình (có hai nghiệm phân biệt, có nghiệm kép,

có nghiệm hoặc vô nghiệm).

3. Áp dụng định lý Viet.

a) Tìm tham số khi biết nghiệm của phương trình.

b) Tìm tham số khi biết dấu của nghiệm (hai nghiệm trái dấu, cùng dấu, cùng dương hoặc

cùng âm)

c) Tìm tham số khi biết hệ thức liên hệ giữa các nghiệm

- Hệ thức đối xứng.

- Hệ thức không đối xứng.

d) Tính giá trị của biểu thức nghiệm theo tham số.

e) Tìm hệ thức độc lập giữa các nghiệm của phương trình không phụ vào tham số.

f) Lập phương trình bậc hai khi biết hai nghiệm của phương trình.

Bài toán tổng quát. Tìm điều kiện để phương trình 2ax +bx c 0 (a 0)

1. Có nghiệm (có hai nghiệm) 0

2. Vô nghiệm 0

3. Nghiệm duy nhất (nghiệm kép, hai nghiệm bằng nhau) 0

4. Có hai nghiệm phân biệt (khác nhau) 0

5. Hai nghiệm cùng dấu 0 và 0P

6. Hai nghiệm trái dấu 0 và 0 0P ac

7. Hai nghiệm dương(lớn hơn 0) 0; S 0 và P 0

Page 23: SỞ GIÁO DỤC VÀ ĐÀO TẠO NAM ĐỊNH

23

8. Hai nghiệm âm(nhỏ hơn 0) 0; S 0 và P 0

9. Hai nghiệm đối nhau 0 và S 0

10.Hai nghiệm nghịch đảo nhau 0 và 1P

11. Hai nghiệm trái dấu và nghiệm âm có giá trị tuyệt đối lớn hơn 0 0P ac và

S 0

12. Hai nghiệm trái dấu và nghiệm dương có giá trị tuyệt đối lớn hơn 0 0P ac và

S 0

1 2

bS x x

a

; 1 2.

cP x x

a

Ví dụ 1. Cho phương trình: 2 - 2 1 – 3 – 0 x m x m

1) Chứng minh phương trình luôn có nghiệm 1 2,x x với mọi m

2) Tìm m để phương trình có hai nghiệm trái dấu

3) Tìm m để phương trình có hai nghiệm cùng âm

4) Tìm m sao cho nghiệm số 1 2,x x của phương trình thoả mãn 2 21 2 10x x .

Hướng dẫn: 1) Ta có 2

2’ 1 15m 1 – –3 – m

2 4m

Do 2

10

2m

với mọi m ;

150

4 0 m

Phương trình luôn có hai nghiệm phân biệt hay phương trình luôn có hai nghiệm

(đpcm)

2) Phương trình có hai nghiệm trái dấu . 0 3 0 3 a c m m

Vậy 3m

3) Theo ý a) ta có phương trình luôn có hai nghiệm

Khi đó theo định lí Viet ta có: 1 2 1. 22 1 ; 3S x x m P x x m

Khi đó phương trình có hai nghiệm âm 2( 1) 0 1

3( 3) 0 3

m mm

m m

Vậy 3m

4) Theo ý a) ta có phương trình luôn có hai nghiệm

Theo định lí Viet ta có: 1 2 1. 22 1 ; 3S x x m P x x m

Khi đó 2 22 2 2

1 2 1 2 1 22 4 1 2 3 4 – 6 10 A x x x x x x m m m m

Theo bài 210 4 – 6 0 2 2 3 0 A m m m m

00 3

32 3 0 2

20 0

02 3 0 3

2

mm

mm m

m mm

mm

Vậy3

2m hoặc 0m

Ví dụ 2. Cho phương trình: 2 2 1 0 x x m ( m là tham số)

Page 24: SỞ GIÁO DỤC VÀ ĐÀO TẠO NAM ĐỊNH

24

1) Phương trình có hai nghiệm là nghịch đảo của nhau

2) Tìm m để phương trình có hai nghiệm 1 2,x x thoả mãn 1 23 2 1x x

Hướng dẫn

a) Ta có ’ 21 – 1 2 –m m

Phương trình có hai nghiệm là nghịch đảo của nhau

' 2 0 20

21 1 21

m mm

m mP

. Vậy 2m

b) Ta có ’ 21 – 1 2 –m m

Phương trình có nghiệm 0 2 – 0 2 *m m

Khi đó theo định lí Viet ta có: 1 2 1 22 1 ; –1 2x x x x m

Theo bài: 1 23 2 1 3x x

Từ (1) và (3) ta có: 2 2 2 4 5 5

1 2 1 2 1 1

3 2 1 3 2 1 2 71 2 1 2 1 2 2

x x x x x x

x x x x x x x

Vậy 34 m là giá trị cần tìm

Ví dụ 3: Cho phương trình bậc hai ẩn x, tham số m : 2 3 0x mx m (1)

1) Giải phương trình với 2.m

2) Gọi 1 2;x x là các nghiệm của phương trình. Tính 2 2 3 31 2 1 2;x x x x theo m .

3/ Tìm m để phương trình có nghiệm 1 3x . Tính nghiệm còn lại.

Hướng dẫn 1) Thay 2m vào phương trình (1) ta có phương trình :

2 22 1 0 ( 1) 0 1 0 1x x x x x

Vậy với 2m phương trình có nghiệm duy nhất 1.x

2) Phương trình : 2 3 0x mx m (1)

2 24( 3) 4 12m m m m

Phương trình có nghiệm 1 2; 0x x

Khi đó theo định lý Vi-et, ta có : 1 2

1 2

( )

3 ( )

x x m a

x x m b

*) 2 2 2 2 21 2 1 2 1 2( ) 2 ( ) 2( 3) 2 6x x x x x x m m m m

*) 3 3 3 3 3 21 2 1 2 1 2 1 2( ) 3 ( ) ( ) 3( 3)( ) 3 9x x x x x x x x m m m m m m

3) Phương trình (1) có nghiệm 2

1 3 ( 3) .( 3) 3 0 2 12 0 6x m m m m

Khi đó : 1 2 2 1 2 26 ( 3) 3x x m x m x x x

Vậy với 6m thì phương trình có nghiệm 1 2 3.x x

BÀI TẬP

Bài 1. Cho phương trình : 2( 1) 2( 1) 2 0m x m x m ( m là tham số).

1) Giải phương trình với 3.m

2) Tìm m để phương trình có hai nghiệm phân biệt 1 2;x x thỏa mãn : 1 2

1 1 3

2x x .

Page 25: SỞ GIÁO DỤC VÀ ĐÀO TẠO NAM ĐỊNH

25

Bài 2. Cho phương trình 2 – 2 – 5 0x x m

1) Giải phương trình với 2.m

2) Tìm m để phương trình có hai nghiệm phân biệt 1 2;x x thỏa mãn 1 22 3 16x x

Bài 3. Cho phương trình 2 1 3 6 0x m x m .

1) Chứng minh rằng phương trình luôn có một nghiệm 1 3x .

2) Tìm giá trị của m để phương trình có nghiệm 2 1 2x .

Bài 4. Cho phương trình: 2 2 1 4 0 1 x m x m ( m là tham số)

1) Giải phương trình (1) với 1.m

2) Tìm các giá trị của m để phương trình (1) có hai nghiệm trái dấu.

3) Với 1 2,x x là nghiệm của (1). Tính theo m giá trị của biểu thức:

1 2 2 11 1 .A x x x x

Bài 5. Cho phương trình: 2 2- 2 - 4 - 3 0x x m m ( với m là tham số). Tìm m để phương

trình có hai nghiệm 1 2,x x thỏa mãn điều kiện 3 31 2 26x x

Bài 6. Cho phương trình: 2 2 2 1 2 0x m x m (với m là tham số)

1) Giải phương trình khi 1m

2) Tìm m để phương trình có hai nghiệm dương?

Bài 7. Cho phương trình: 2 24 4 2 0 x mx m m . Tìm các giá trị của m để phương

trình có 2 nghiệm phân biệt 1 2,x x thỏa mãn 1 2 2x x .

Bài 8. Cho phương trình: 2 21 2 0x m x m m (với m là tham số)

1) Giải phương trình với 2.m

2) Tìm m để phương trình có 2 nghiệm 1 2,x x sao cho biểu thức 2 21 2A x x đạt giá

trị nhỏ nhất

Bài 9. Cho phương trình : 22 3 0 1x mx m

1) Giải phương trình (1) khi 1m .

2) Chứng minh rằng phương trình (1) luôn có hai nghiệm phân biệt với mọi giá trị

của m .

3) Tìm các giá trị của m để phương trình (1) có hai nghiệm trái dấu và nghiệm âm có

giá trị tuyệt đối lớn hơn nghiệm dương.

Bài 10. Cho phương trình 2 – 2 – 2 0 x mx ( m là tham số)

1) Giải phương trình khi 1

2m .

2) Chứng minh rằng với bất kì giá trị nào của m thì phương trình đã cho luôn có

nghiệm. Hãy xác định m để phương trình có hai nghiệm 1 2,x x thỏa mãn 1 2 1 1.x x

Bài 11. Cho phương trình ẩn x, tham số m : 2 5 ( 5) 0 (1)mx x m

1) Giải phương trình (1) khi 5.m

2) Chứng tỏ rằng phương trình (1) luôn có nghiệm với mọi giá trị của m .

3) Trong trường hợp phương trình (1) có hai nghiệm phân biệt 1 2,x x . Hãy tính theo

m giá trị của biểu thức 2 21 2 1 216 3( ).A x x x x Tìm m để 0.A

Bài 12. Cho phương trình ẩn x, tham số m:

Page 26: SỞ GIÁO DỤC VÀ ĐÀO TẠO NAM ĐỊNH

26

2 2 3( 3) 2( 3 ) 12 0 (1)m x m m x m

1) Tìm số nguyên m nhỏ nhất sao cho phương trình (1) có hai nghiệm phân biệt.

2) Gọi 1 2,x x là hai nghiệm của phương trình (1). Tìm số nguyên m lớn nhất sao cho 2 21 2x x là một số nguyên.

Bài 13. Cho các số , ,a b c thỏa điều kiện:

2

0

2

c

c a ab bc ac

Chứng minh rằng phương trình 2 0 ax bx c luôn luôn có nghiệm.

Bài 14. Cho , ,a b c là các số thực thỏa điều kiện 2 0. a ab ac

Chứng minh rằng phương trình 2 0 ax bx c có hai nghiệm phân biệt.

Bài 15. Cho ,a b là hai số thực thỏa mãn điều kiện 1 1 1

2a b . Chứng minh rằng ít nhất

một trong hai phương trình sau phải có nghiệm: 2 0 x ax b và 2 0.x bx a

Chuyên đề 3

HỆ PHƯƠNG TRÌNH

A. Hệ phương trình cơ bản

I. Hệ phương trình có chứa 1 phương trình bậc nhất:

1

; 0 2

ax by c

F x y

Phương pháp giải: Từ phương trình 1 rút một ẩn theo ẩn còn lại rồi thế vào phương trình

2

II. Hệ phương trình đối xứng loại I:

; 0 1

; 0 2

F x y

G x y

trong đó

; ;

; ;

F x y F y x

G x y G y x

Đổi vai trò của ,x y cho nhau thì phương trình 1 , 2 không thay đổi.

Phương pháp giải:

+) Đặt .

S x y

P x y

điều kiện 2 4S P quy hệ phương trình về 2 ẩn ,S P

+) Tìm , ,S P x y

III. Hệ phương trình đối xứng loai II:

; 0 1

; 0 2

F x y

G x y

trong đó

; ;

; ;

F x y G y x

G x y F y x

Đổi vai trò của ,x y cho nhau thì phương trình 1 trở thành phương trình 2 và phương

trình 2 trở thành phương trình 1 .

Phương pháp giải: Trừ vế với vế hai phương trình của hệ ta được một phương trình có dạng

0; 0

; 0

x yx y f x y

f x y

.

Page 27: SỞ GIÁO DỤC VÀ ĐÀO TẠO NAM ĐỊNH

27

IV. Hệ có phương trình đẳng cấp:

1 . .... 0 1

; 0

n n k k nk na x a x y a y

F x y

Tổng số mũ của x và y trong từng số hạng của phương trình 1 cùng bằng n

Từ đó ta xét hai trường hợp: + 0y thay vào để tìm x

+ 0y ta đặt x ty thì thu được phương trình: 1 .... 0n n kk na t a t a

+ Giải phương trình tìm t sau đó thế vào hệ ban đầu để tìm ,x y

Chú ý: ( Ta cũng có thể đặt y tx )

V. Một số phương pháp thường sử dụng khi giải hệ phương trình 1. Phương pháp thế, phương pháp cộng 2.Phương pháp đặt ẩn phụ 3.Phương pháp đánh giá … B. Ví dụ minh họa

Ví dụ 1: Giải hệ phương trình 2

3 03

1

xy

y

x y

Lời giải: Điều kiện 3.y

HPT

2

2 2

9 5 5

21 4

x y x x

yx y y

Vậy hệ có 4 nghiệm ;x y là 5;2 , 5;2 , 5; 2 , 5; 2 .

Ví dụ 2: Giải hệ phương trình

44

1

2 5 3

1

x x

x y

x x y

Lời giải: Điều kiện 1; 0.x y

HPT

44

1

2 53

1

x x

x y

x x

x y

Đặt 1

xu

x

xv

y

Hệ phương trình trở thành

174 4 18

2 5 3 2

9

uu v

u vv

Hay

1717

1 181532

29

xx

x

x y

y

Vậy hệ có nghiệm ;x y là 153

17; .2

Ví dụ 3. Giải hệ phương trình 1 1

2 1 8

x y

x y x

Lời giải :Điều kiện: 1; 0.x y

Page 28: SỞ GIÁO DỤC VÀ ĐÀO TẠO NAM ĐỊNH

28

HPT 1 1

1 2 1 9

x y

x y x

Đặt 1

0, 0u x

u vv y

Hệ phương trình trở thành 2 2 2

2

11 1

2 9 2 8 2(lo¹i)

3

u

vu v v u

u v u u u

v

Hay 1 2 3

.11

x x

yy

Vậy hệ có nghiệm ;x y là 3;1 .

Ví dụ 4. Giải hệ phương trình

2

2

1 1

2 8 0 2

xy x y

x xy

Lời giải: Ta có 1

1 1 1 1 0 1 1 01

yx y y y y x y

y x

Với 1y từ 2 ta có 2 2 8 0x x (vô nghiệm)

Với 1y x từ 2 ta có 2 2 2 32 1 8 0 2 8 0

4 3

x yx x x x x

x y

Vậy hệ có nghiệm ;x y là 2;3 , 4; 3 .

Ví dụ 5: Giải hệ phương trình 2 2

2 51

5

xy yy

x

x y

Lời giải: Điều kiện: 0.x

HPT

22 2

55

5 2 5

x y xyxy x y

x y x y xy

Đặt 2 4

x y SS P

xy P

Hệ phương trình đã cho trở thành

2 2

3

25 5

2 5 2 15 0 5¹

10

S

PS P P S

S P S S Slo i

P

Khi 3

,2

S

P

ta có

3 1

2 2

x y x

xy y

hoặc

2.

1

x

y

Vậy hệ có 2 nghiệm ;x y là 2;1 , 1;2 .

Ví dụ 6. Giải hệ phương trình 2

2

2

2

x x y

y y x

Lời giải: Điều kiện: , 0x y . Trừ hai phương trình của hệ cho nhau ta thu được:

2 2 2 1 1 2 0x x y y y x x y x y x y x y

Vì 1 2 0 1x y x y x y x y

Page 29: SỞ GIÁO DỤC VÀ ĐÀO TẠO NAM ĐỊNH

29

Hay 2 2

0

2 0 2 1 1 0 1

3 5

2

x

x x x x x x x x x x x

x

Vậy hệ có 3 nghiệm: 3 5 3 5

; 0;0 , 1;1 , ;2 2

x y

Ví dụ 7: Giải hệ phương trình3

2 2

2 0

2

y x y

x y

Lời giải

HPT 3 2 23 3 2 2 3

2 22 2 2 2

4 04 2 0 3 0

22 2

y x y x yy x y y x y xy x

x yx y x y

Ta có 0; y không là nghiệm của hệ phương trình

Xét 0x đặt y tx

Hệ phương trình đã cho trở thành

3 2 3 3 2

2 22 2

3 1 0 3 1 1 1

11 21 2

t t t x t t t t

xx tx t

Vậy hệ có 2 nghiệm ;x y là 1;1 , 1; 1

Ví dụ 8: Giải hệ phương trình

3 2 33 6 4 3 1

3 1 3 2

x x x y y

x y

Lời giải: Điều kiện: 3, 1.x y

Ta có 3 31 1 3 1 3 .x x y y Đặt 1x a phương trình 1 trở thành

2 22 2

3 3 3 33 3 3 0 Do 3 0

2 4 2 4

y y y ya a y y a y a a y a

Hay 1y x thay vào phương trình 2 ta được

1 1

3 3 3 1 2 0 4 0 4 33 2

x x x x x x yx x

Vậy hệ có 1 nghiệm ;x y là 4;3

Ví dụ 9: Giải hệ phương trình

2 2

2 2

2 4 3 5 0 1

6 8 0 2

x y xy x y

x y x

Lời giải:

Ta có 2 21 2 2 3 5 0x y x y y (coi x là ẩn, y là tham số)

2 2' 2 3 5 1.y y y y Suy ra phương trình 1 có nghiệm 1 3y

Page 30: SỞ GIÁO DỤC VÀ ĐÀO TẠO NAM ĐỊNH

30

2 22 3 1x y

2

2

4 23 1

1 1 41

xx

yy

Từ 3 , 4 suy ra ;x y là nghiệm của hệ thì 1y .Thay 1y vào hệ ta được

2

2

6 9 03

6 9 0

x xx

x x

Vậy hệ có 1 nghiệm ;x y là 3;1

Ví dụ 10. Giải hệ phương trình

2 2

1 3 1 3 2 1

4 4 7 0 2

x y y x

x y x y

Lời giải Ta có

2

2 22 2

2

2 1 1 32 4 4 7 0 2 2 1

1 32 1

x xx y x y x y

yy

Suy ra 1 0 2

1 33 0 2

x x yx y

y

1 0 21 3

3 0 2

y y xy x

x

Do dó ta có 2 2

1 3 1 3 22 2

x y y xx y y x

Hay 1 4y x thay vào phương trình 2 ta được

2

4 2 4 2

1 2 22

2 4 2 4 2

2 2

x y

x

x y

Vậy hệ có 2 nghiệm ;x y là 4 2 4 2 4 2 4 2

; , ;2 2 2 2

Chuyên đề 4

PHƯƠNG TRÌNH CHỨA CĂN

I. Phương pháp khai căn 1. Kiến thức cơ bản

2

khi 0

khi 0

f x f xf x f x

f x f x

33 f x f x

2. Ví dụ minh họa

Ví dụ :Giải phương trình sau 24 2 4 2 2 1x x

Lời giải: Điều kiện: 2 2.x

2

1 2 2 2 1 2 1 2 1 1x x x x x x

Page 31: SỞ GIÁO DỤC VÀ ĐÀO TẠO NAM ĐỊNH

31

Kết hợp điều kiện phương trình có tập nghiệm là 1 .S

II. Phương pháp nâng lũy thừa 1. Kiến thức cơ bản

0 0)

g x f xf x g x

f x g x

2

0)

g xf x g x

f x g x

3 3) f x g x f x g x

33) f x g x f x g x

2. Ví dụ minh họa

Ví dụ 1:Giải phương trình 2 2 4 3 4 1x x x

Lời giải: Ta có

222

4

4 33 4 01 2.3 5

2 4 3 48 26 20 0 4

2

xx x

xxx x x

x x

x

Kết hợp điều kiện phương trình có tập nghiệm là 2 .S

Ví dụ 2: Giải phương trình 2 7 2 1 2 2 2x x x

Lời giải: Điều kiện:1

22

x

2

2

2

2 2 7 2 2 2 1 2 7 8 4 2 1 2 5 2

10

2 5 2 23 5 2 0

3

x x x x x x x x

xx

x x xxx x

Kết hợp điều kiện phương trình có tập nghiệm là 2

;1 .3

S

III. Phương pháp đưa phương trình về dạng tích

1. Kiến thức cơ bản:

0. 0

0

f xf x g x

g x

2. Ví dụ minh họa

Ví dụ 1: Giải phương trình: 26 2 9 3 3 0 1x x x

Lời giải: Điều kiện: 3 3.x

Page 32: SỞ GIÁO DỤC VÀ ĐÀO TẠO NAM ĐỊNH

32

2

2

1 3 3 3 3 0 3 3 3 3 1 0

3 3 1.1

3 1 3 1.2

1.1 3 3 0

111

2.1 2 3 2 1 22

4 11

x x x x x x x x

x x

x x

x x x

xx x x

x

Kết hợp điều kiện phương trình có tập nghiệm là 11

0; .2

S

Ví dụ 2: Giải phương trình 2 4 3 2 2 0 2x x x

Lời giải: Điều kiện: 2x

2

1 2 3 2 3 2 0 2 3 2 2 3 0

2 3 2.12 3 2 0

2 2.2

2.1 7

02.2 2

2 0

x x x x x x x x

xx x x

x x

x

xx

x x

Kết hợp điều kiện phương trình có tập nghiệm là 2;7 .S

IV. Phương pháp sử dụng đại lượng liên hợp 1. Kiến thức cơ bản

2

3 3 3 32 2 2 23 3 3 3 3 3

3 32 2 3 2 2 33 3 3 3

) 0, 0 ) 0

) )

) )

A B A B A B A B A B A B A B A

A B A AB B A B A B A AB B A B

A B A B A B A B A B A B A B A B

2. Ví dụ minh họa

Ví dụ : Giải phương trình 2 2 2 1 2 12 0x x x

Lời giải: Điều kiện: 1.x

2 12 16 161 2 2 2 12 4 1 0 2 2 0

2 12 4 1

272 1 0

2 12 1 7 1.12 12 1

x xx x x x

x x

xx

x xx x

Page 33: SỞ GIÁO DỤC VÀ ĐÀO TẠO NAM ĐỊNH

33

2

2

38

1.1 2 2 10 12 38 3 134 4 10293

268 1492 0

xx x x x

x x

Ví dụ 2. Giải phương trình 2 1 3 5 .x x x

Lời giải: Điều kiện 5

.3

x

Phương trình đã cho tương đương với: 2 2 2 1 3 5 0x x x

1 3 5 3 5 1 2 1 3 5 0x x x x x x

1 3 5 3 5 1 2 0x x x x

1 3 5 0

3 5 1 2 0

x x VN

x x

Lại có: 3 5 1 2 0x x 4 2 1x x 2

4

12 20 0

x

x x

10x

Kết luận. Nghiệm của phương trình đã cho là 10.x

Ví dụ 3. Giải phương trình 2 12 4 66 2x x x

Lời giải: Điều kiện: 4.x

22 30 12 4 3 0 2

512

5 6 0 126 0 2.14 3

4 3

x x x

x

x xxx

x

Ta có với mọi

6 2

4 120

4 3

x

x

x

Phương trình 2.1 vô nghiệm

Kết hợp điều kiện phương trình có tập nghiệm là 5 .S

Ví dụ 4. Giải phương trình 33 5 3 4 4x x

Lời giải: Điều kiện 3.x 33 5 3 4x x 33 2 5 3 2 0x x

2 33

5 110

3 2 5 3 2 5 3 4

xx

x x x

2 33

1 51 0

3 2 5 3 2 5 3 4x

x x x

1x

2 33

1 5 0, 3

3 2 5 3 2 5 3 4Do x

x x x

Kết luận. Phương trình có nghiệm 1.x

Ví dụ 5. Giải phương trình 23 1 5 4 3 3x x x x

Lời giải: Điều kiện: 1

.3

x Ta có 23 1 3 1 2 5 4 0x x x x x x

Page 34: SỞ GIÁO DỤC VÀ ĐÀO TẠO NAM ĐỊNH

34

Ta có với mọi 1 3 1 01

.3 2 5 4 0

x xx

x x

Do đó 2 2

23 01 3 1 2 5 4

x x x xx x

x x x x

2

2

1 11 0

1 3 1 2 5 4

00

1 111 0

1 3 1 2 5 4

x xx x x x

x xx

xVNx x x x

Kết hợp điều kiện phương trình có tập nghiệm là 0;1 .S

3. Bài tập tự luyện Giải các phương trình sau

a/ 2 2 2 1 1 2x x x x x b/

2 3 3 22 2 1 (1 ) (1 ) 2 1x x x x

c/ 22 6 1 1x x x d/ 4 3 10 3 2x x

e/ 4 1 2 1x x x h/ 22 7 2 1 8 7 1x x x x x i/

22( 2) 1 1x x x

k/ 3 2 2 ) 2 6x x x . l/ 23 1 6 3 14 8 0x x x x

m/ 2 21 ( 2) 2 2x x x x x n/

3 2 22 6 1 2 1 1 3 1.x x x x x x x

B.Phương pháp đặt ẩn phụ I. Đặt ẩn phụ đưa phương trình về dạng một ẩn

1. Đưa phương trình về dạng 2 0at bt c 1.1 Ví dụ minh họa

Ví dụ 1: Giải phương trình 2 1 2 1

122 2

x x

x x

Lời giải: Điều kiện 2 2, 0.x x

2

2 2 2 22 4 12 4 4 4 2 0

2 2

x xx x x x x x x

x

Đặt 24 0 2t x t

Phương trình trở thành 2

12 0

2

t loait t

t

Với 2t ta có 24 2 0x x

Kết hợp điều kiện phương trình có tập nghiệm là 0 .S

Ví dụ 2: Giải phương trình 26 2 3 3 3 1 4 6x x x x x 2 (Đề TS LHP NĐ

2018)

Page 35: SỞ GIÁO DỤC VÀ ĐÀO TẠO NAM ĐỊNH

35

Lời giải: Điều kiện: 2 3x

2

2 3 2 2 3 4 2 4 2 3 3 10

2 2 3 3 2 2 3 10 0

x x x x x x

x x x x

Đặt 2 2 3 0t x x t

Phương trình trở thành 2

2 ¹3 10 0

5

t lo it t

t

Với 5,t ta có 2 2 3 5 2.x x x

Kết hợp điều kiện phương trình có tập nghiệm là 2 .S

2.1 Đưa phương trình về dạng . .a f x bg x c f x g x

Ví dụ : Giải phương trình 2 33 1 3 1x x x 3

Lời giải: Điều kiện: 1x

2 2

2 2

1 13 2 1 1 3 1 1 2 3 1 0

1 1

x xx x x x x x

x x x x

Đặt 2

10

1

xt t

x x

Phương trình trở thành 2

1

2 3 1 0 1

2

t

t tt

+) Với 1

2t ta có 2

2

1 13 5 0

1 4

xx x

x x

(vô nghiệm)

+) Với 1t ta có 2

2

11 2 2

1

xx x

x x

Kết hợp điều kiện phương trình có tập nghiệm là 2 .S

3.1 Đưa phương trình về dạng 2 2. .a f x bg x cf x d g x

Ví dụ : Giải phương trình 22 5 2 3 1 1x x x x

Lời giải: Điều kiện: 0x

2

21 2 1 3 1 3 2 3 3

11

x xx x x x

xx

Đặt 01

xt t

x

. Phương trình trở thành: 2

2

1 12 3 3

28 18 7 0

tt t t

t t

Với 1

,2

t ta có 21

1 1 1 11 2

xx x x

x

Kết hợp điều kiện phương trình có tập nghiệm là 1 .S

II. Đặt ẩn phụ đưa phương trình về dạng hai ẩn 1. Đưa về phương trình 2 ẩn đồng bâc

Bậc 2: 2 2. . . . 0a u b u v c v

Bậc 3: 3 2 2 3. . . . . 0au b u v c u v d v

Page 36: SỞ GIÁO DỤC VÀ ĐÀO TẠO NAM ĐỊNH

36

Ví dụ 1:Giải phương trình 2 22 6 2 1 2 0.x x x x

Lời giải: Điều kiện 1.x Đặt 2 2 ;x a 1x b , 0a b

Ta có: 2 22 6 0a ab b 2 3 2 0a b a b 2 3

2

a b

a b

Với 2 3 ,a b ta có: 22 2 3 1x x 24 9 1 0x x 9 97

8

x

Nghiệm của phương trình đã cho là 9 97

.8

x

Ví dụ 2:Giải phương trình 2 2 24 5 1 3 8 5 1x x x x x x

Lời giải: Điều kiện 1.x Phương trình đã cho tương đương với:

2 2 2 24 1 1 8 1 1 1 1 3 1 1 0 *x x x x x x x x

Đặt 2

1

1

x u

x v

0, 0u v . Phương trình đã cho trở thành:

3 2 2 34 8 3 0u u v uv v 2 2 3u v u v u v

2

2 3 0

0

u v

u v loaïi

u v loaïi

Khi 2 ,u v ta có 22 1 1x x 2

1

4 3 0

x

x x

2 7x

Vậy các nghiệm của phương trình đã cho là 2 7x

2) Đưa phương trình về phương trình về dạng f u f v ( hai ẩn đối xứng)

Ví dụ 1. Giải phương trình 2 22 3 2 4 1 1x x x x

Giải Lời giải: Điều kiện 2.x

2 21 1 1 2 4 2 4x x x x

Đặt 2 1;u x u,v 0 , phương tình đã cho trở thành: 2 2 2 2u u v v u v u v 1 0u v u v u v

Khi u v , ta có: 2 1 2 4x x 2 2 3 0x x 1

3

x

x

Vậy các nghiệm của phương trình là 0; 2.x x

Ví dụ 2. Giải phương trình 3 2 5 3 5 3 5 3x x x x x

Lời giải: Điều kiện 5

.3

x

3

32 2 5 3 2 5 3x x x x

Đặt 5 3u x u 0 , phương tình đã cho trở thành:

v 2x 4,

Page 37: SỞ GIÁO DỤC VÀ ĐÀO TẠO NAM ĐỊNH

37

3 3 2 22 2 2 0x x u u x u x xu u

2 232 0

2 4

x u

u ux VN

Khi x u , ta có: 2

0 3 295 3

23 5 0

xx x x

x x

Vậy các nghiệm của phương trình là 3 29

2x

3.Đưa phương trình về phương trình bặc hai hai ẩn ( 1 ẩn coi như tham số)

Ví dụ 1: Giải phương trình sau 2 21 2 3 1 1x x x x

Lời giải: Đăt t = 2 2 3 1x x t

Phương trình trở thành : 2 ( 1) 2 2 0 1.1t x t x (Đặt ẩn phụ không hoàn toàn)

22 6 9 3x x x “Delta chính phương”

Khi đó 1

1.12

t x

t

Khi 2,t ta có 2 22 3 2 2 1 0 1 2x x x x x

Khi 1t x ta có 2 2 3 1x x x VN

Vậy tập nghiệm của phương trình đã cho là 1 2S

Ví dụ 2 : Giải phương trình 22 2 1 1 1 3 1 2x x x x

Lời giải: Điều kiện 1 1x

Đặt 1 , 1 , 0a x b x a b

Phương trình đã cho trở thành: 2 24 2 – 2a b ab a b

2 22 (4 ) 2 0 2.1a b a b b

Ta có 2 2 2(4 ) 8 16 (3 4)b b b b 2.1 2

2

ba

a b

Khi ,2

ba ta có

1 31 1

2 5x x x

Khi 2 ,a b ta có 1 2 1x x 1 1 2x x 1 1 0x x (thỏa

mãn)

Vậy phương trình đã cho có tập nghiệm 3

;05

S

.

III. Đặt ẩn phụ đưa về hệ phương trình

Ví dụ 1: Giải phương trình 32 3 2 3 6 5 8 0x x (Đưa về hệ giải được bằng phương pháp thế)

Page 38: SỞ GIÁO DỤC VÀ ĐÀO TẠO NAM ĐỊNH

38

Lời giải: Điều kiện 6

.5

x Đặt 3 3 2 ;x a 6 5x b 0 ,b ta có hệ: 3 2

2 3 8

5 3 8

a b

a b

3 2

8 2

3

15 4 32 40 0

ab

a a a

2

4

a

b

Khi

2,

4

a

b

ta có 2.x

Vậy phương trình đã cho có tập nghiệm 2S

Ví dụ 2: Giải phương trình 1 1

22x x

(TH&TT) (Đưa về hệ có phương pháp giải)

Lời giải: Điều kiện 0 2x .

Đặt 2

x a

x b

2 2 2a b , 0a b

Ta có hệ phương trình:

2 2 2

1 12

a b

a b

22 2

2

a b ab

a b ab

2

2 1 0

2

ab ab

a b ab

1

2

1

ab

a b

(loại) hoặc 1

2

ab

a b

+ Ta có 1

1.2

aba b

a b

Khi đó

11

2 1

xx

x

Vậy tập nghiệm của phương trình đã cho là 1 .S

IV. Bài tập tự luyện Bài 1:Giải các phương trình sau

a/ 2 23 2 2 3x x x x b/1 3

21 5

xx

x x

c/ 2 34 6 3 13x x x d/

2

36 2

9

xx

x

e/ 23 2 1 4 9 2 3 5 2x x x x x g/

22 5 3 2 3 1 2 27.x x x x x

h/ 2 26 28 2 11 2 1 .x x x x i/ 2 25 4 3 18 5x x x x x

k/ 3 2 22 6 6 1.x x x x x x l/ 2 28 2 5 6 2 1 3 2 1.x x x x x

m/ 3 2 234 5 6 7 9 4.x x x x x n/

1 1 5 2 1 2 1 2 1.x x x x x

B i 2: Gi i các ph ng trình sau

a/ 3 2 1 1.x x b/ 2 24 2 3 4 .x x x x

c/ 2 5 5x x d/ 22 6 1 4 5.x x x

C. Phương pháp đánh giá 1. Kiến thức cơ bản

Page 39: SỞ GIÁO DỤC VÀ ĐÀO TẠO NAM ĐỊNH

39

Dạng 1:

2 2

00

0

f xf x g x

g x

Dạng 2: f x g x

Nếu

f x M

g x M

thì

f x Mf x g x

g x M

2. Ví dụ minh họa

Ví dụ 1: Giải phương trình 13 1 9 1 16 (1)x x x

Lời giải: Điều kiện 1x

Cách 1: 2 2

11

1 3 5213 1 3 1 032 2 4

14

x

x x x

x

Cách 2: Ta có

1 313 1 13.2. 1 13

4 413 1 9 1 16

9 139 1 3.2 1 3

4 4

x x x

x x x

x x x

Do đó phương trình

11

5419 4

14

x

x

x

Vậy tập nghiệm của phương trình đã cho là 5

.4

S

Ví dụ 3 : Giải phương trình 2 2 21 1 4 2x x x x x x

Giải:Điều kiện: 1x

Ta có 2 24 2 2 2(1 ) 2x x x Dấu bằng xảy ra khi và chỉ khi x =1

Áp dụng bđt cosi cho hai số 2 21; 1x x x x không âm

2 2 2 2 2 21 1 2 1 1 2 ( 1) 2x x x x x x x x x x

Dấu ‘’=’’ xảy ra khi và chỉ khi 2 21 1 1x x x x x

Khi đó phương trình đã cho tương đương với

2

2 2

4 2 2

1 1 2

x x

x x x x

1x

Vậy tập nghiệm của phương trình đã cho là 1S

Ví dụ 3 : Giải phương trình 3 23 1 5 2 10 34 40x x x x x x

Giải :Điều kiện:5

12

x .

Page 40: SỞ GIÁO DỤC VÀ ĐÀO TẠO NAM ĐỊNH

40

2 2

2 2 2

3 1 5 2 (4 )( 6 10) 3 1 5 2 (3 ) 1 4

3 1 1. 5 2 (3 ) 1 ( 1) ( 5 )

x x x x x x x x x x x

x x x x x x

Áp dụng bất đẳng thức Bunhia

2 22

3 1 1. 5 2 3 1 1 5x x x x x x

Do đó phương trình đã cho 2

7

3 1. 5 2 3 13 14 0 3

2

xx x x x x

x

Vậy tập nghiệm của phương trình đã cho là 7

;2 .3

S

3. Bài tập tự luyện Giải các phương trình sau

a/ 4 2 2 22 2 16 2 6 20 0x x x x x x b/3 2 2 3 29 18 36 9 9.x x x x x

c/2

3 3 14 .

1

x x

x x x

d/ 24 6 10 27x x x x

Chuyên đề 5

HÌNH HỌC PHẲNG

A. Kiến thức cơ bản

1. Đường tròn, sự xác định đường tròn

2. Vị trí tương đối của đường thẳng với đường tròn, của hai đường tròn.

3. Tính chất của dây cung và đường kính.

4. Tiếp tuyến của đường tròn: Định nghĩa, các tính chất và các phương pháp chứng

minh.

5. Góc và đường tròn: góc ở tâm; góc nội tiếp; góc có đỉnh ở trong, ngoài đường

tròn; góc giữa tiếp tuyến và một dây cung: Định nghĩa, các tính chất.

6. Tứ giác nội tiếp: Định nghĩa, các tính chất, phương pháp chứng minh một tứ giác

là tứ giác nội tiếp.

7. Dấu hiệu nhận biết các hình: hình bình hành, hình thang, hình thang cân, hình

thoi,hình chữ nhật, hình vuông.

8. Các tính chất của trực tâm tam giác, tâm đường tròn nội tiếp tam giác, tâm đường

tròn ngoại tiếp tam giác, trọng tâm tam giác.

9. Các hệ thức lượng trong tam giác; tỉ số lượng giác của góc nhọn.

10. Công thức tính diện tích các hình, diện tích hình tròn, hình quạt, hình viên phân.

B. Một số ví dụ áp dụng

Ví dụ 1. (Đề thi tuyển sinh vào trường THPT chuyên Lê Hồng Phong năm học 2013 -

2014 môn Toán chung)

Page 41: SỞ GIÁO DỤC VÀ ĐÀO TẠO NAM ĐỊNH

41

Cho tam giác ABC nhọn. Nửa đường tròn đường kính AB cắt các đoạn thẳng ,CA CB

theo thứ tự tại ,M N (khác ,A B ). Gọi H là giao điểm của AN và BM .

1. Chứng minh tứ giác CMHN là tứ giác nội tiếp và 090BAC ANM .

2. Gọi ( )O là đường tròn ngoại tiếp tam giác ABC . Kẻ đường kính CD của đường tròn

( )O . Chứng minh AH BD .

3. Gọi I là trung điểm của AB . Đường thẳng qua H vuông góc với IH lần lượt cắt các

đường thẳng ,CA CB tại ,P Q . Chứng minh H là trung điểm của PQ.

Hướng dẫn

1) Có 0 090 ; 90ANB AMB (*) (góc nội tiếp chắn

nửa đường tròn đường kính AB)

0 090 ; 90CNH CMH

4 điểm C, M, N, H thuộc đường tròn đường kính

CH

tứ giác CMHN nội tiếp.

Vì M và N thuộc đường tròn đường kính AB nên tứ

giác AMNB nội tiếp 0180BAC BNM

Mà 0180CNM BNM BAC CNM (1)

Mặt khác 090CNM ANM CNA (2).

Từ (1) và (2) 090BAC ANM .

2) Từ (*) ,AH BC BH AC (3)

Mặt khác CD là đường kính của đường tròn (O)

0 090 ; 90 ;CBD CAD BD BC AD AC (4).

Từ (3) và (4) / / ; / /AH BD BH AD tứ giác AHBD là hình bình hành. AH BD .

3) Theo chứng minh trên tứ giác AHBD là hình bình hành mà I là trung điểm của đường

chéo AB H, D, I thẳng hàng.

090DH PQ PHD , mặt khác P thuộc AC và 090AD AC DAP .

Vậy 4 điểm A, H, D, P thuộc đường tròn đường kính PD.

Xét trên đường tròn đường kính PD có P và A nằm về cùng một phía đối với HD

HPD HAD , mà AHBD là hình bình hành HAD HBD HPD HBD (5)

Tương tự có 090QHD QBD 4 điểm H, Q, B, D thuộc đường tròn đường kính DQ

HQD HBD (6). Từ (5) và (6) HPD HQD tam giác DPQ cân tại D có DH là

đường cao nên H là trung điểm của PQ.

Ví dụ 2. Cho tam giác ABC nhọn AB AC nội tiếp đường tròn ,O AD là đường kính

của O . Tiếp tuyến tại D của O cắt tia BC tại .M Đường thẳng MO cắt ,AB AC lần

lượt tại E và F .

1. Chứng minh rằng 2 .MD MC MB .

Q

P

I

D

H

N

M

O

C

A B

Page 42: SỞ GIÁO DỤC VÀ ĐÀO TẠO NAM ĐỊNH

42

2. Gọi H là trung điểm của đoạn .BC Chứng minh rằng tứ giác MDHO nội tiếp.

3. Qua B kẻ đường thẳng song song với MO cắt đoạn AD tại .P Chứng minh rằng

tứ giác BHPD nội tiếp.

4. Chứng minh rằng O là trung điểm của đoạn .EF

Hướng dẫn:

1. Vì MD là tiếp tuyến của đường

tròn O nên ta có MDC MBD

Xét MDC và MBD có

MDC MBD và M chung

Do đó MDC ∽ MBD

MD MC

MB MD 2 .MD MB MC

2. Vì H là trung điểm của BC nên

ta có 090OH BC OHM

Vì MD là tiếp tuyến của đường tròn

O nên 090MD OM ODM

Ta có 090OHM ODM

Suy ra 4 điểm , , ,O H D M cùng thuộc đường tròn đường kính OM hay tứ giác MDHO nội

tiếp.

3. Vì tứ giác MDHO nội tiếp nên OMH ODH

Vì ||OM BD nên OMH MBD (2 góc so le trong)

Do đó MBD ODH hay HBD PDH . Suy ra tứ giác BHPD nội tiếp.

4. Vì tứ giác BHPD nội tiếp nên 0180MHP BHP BDP BDA BCA . Suy ra

||HP AC

Gọi K là giao điểm của BP và AC . Xét tam giác BCK có ||HP CK

Mà H là trung điểm của đoạn BC nên P là trung điểm của đoạn CK hay PB PK (1)

Xét tam giác ABK ta có ||EF BK nên theo định lý Ta - let ta có OE AO OF

BP AP PK (2)

Từ (1) và (2) suy ra OE OF hay O là trung điểm của đoạn .EF

Ví dụ 3. Từ một điểm A bên ngoài đường tròn O vẽ hai tiếp tuyến ,AB AC với O (

,B C là các tiếp điểm). Kẻ đường kính CD của O . AD cắt O tại điểm thứ hai là M .

Gọi H là giao điểm của AO và BC .

1. Chứng minh rằng tứ giác AMHC nội tiếp.

2. Gọi N là giao điểm của BM và .AO Chứng minh rằng N là trung điểm của đoạn

.AH

H

K

P

E

F

M

D

O

A

BC

Page 43: SỞ GIÁO DỤC VÀ ĐÀO TẠO NAM ĐỊNH

43

3. Gọi ,I K lần lượt là giao điểm của đường thẳng AO với O . Chứng minh rằng

1 1 1.

AN AI AK

Hướng dẫn: 1. Vì ,AB AC là các tiếp tuyến của

đường tròn O nên OA BC hay

090AHC

Mặt khác vì CD là đường kính của

O nên 0 090 90CMD AMC

Ta có 090AHC AMC nên 4

điểm , , ,A H M C cùng thuộc đường

tròn đường kính AC hay tứ giác

AMHC nội tiếp.

2. Vì tứ giác AMHC nội tiếp nên

MAH MCH (1)

Vì AB là tiếp tuyến của đường tròn O nên ABM BCM MCH (2)

Từ (1) và (2) suy ra MAH ABM hay MAN ABN

Xét NAM và NBA có MAN ABN và N chung nên NAM NBA ∽

2 .NA NM

AN NM NBNB NA

(3)

Mặt khác Vì tứ giác AMHC nội tiếp nên MHA MCA (4)

Vì AC là tiếp tuyến của đường tròn O nên ACM CBM (5)

Từ (4) và (5) suy ra MHA CBM hay MHN HBN

Xét NHB và NMH có MHN HBN và N chung nên NHB NMH ∽

2 .NH NB

HN NM NBNM NH

(6)

Từ (5) và (6) suy ra AN HN hay N là trung điểm của đoạn .AH

3. Gọi R là bán kính của O

Tam giác ABO vuông tại B, đường cao BH nên suy ra 2 .AB AH AO

Vì AB là tiếp tuyến của đường tròn O nên ABI AKB (7)

Do đó 2 .AB AI

ABI AKB AB AI AKAK AB

∽ (7)

Kết hợp (7), (8) và kết quả câu 2 ta có

2

1 1 2 2 1

. .

AI AK AO R AO R AO

AI AK AI AK AB AH AO AH AN

Ví dụ 4. Cho nửa đường tròn tâm O đường kính 2 .AB R Gọi C là trung điểm đoạn .OA

Tia Cx vuông góc với AB cắt nửa đường tròn tại .I Lấy K là ddierm bất kì trên đoạn

N

M

D

KHIA O

B

C

Page 44: SỞ GIÁO DỤC VÀ ĐÀO TẠO NAM ĐỊNH

44

thẳng CI , ,K C K I . Tia AK cắt nửa đường tròn đã cho tại .M Tiếp tuyến với nửa

đường tròn tâm O tại M cắt Cx tại N , tia BM cắt Cx tại .D

1. Chứng minh rằng tứ giác ACMD nội tiếp.

2. Chứng minh rằng tam giác MNK cân.

3.Tính diện tích tam giác ABD theo R khi K là trung điểm đoạn .CI

4. Chứng minh rằng khi K di động trên đoạn CI thì tâm của đường tròn ngoại tiếp

tam giác AKD luôn nằm trên một đường thẳng cố định.

Hướng dẫn:

1. Ta có 090ACD AMD nên 4 điểm

, , ,A C M D cùng thuộc một đường tròn

Hay tứ giác ACMD nội tiếp.

2. Ta có 090BCK BMK nên tứ giác BCKM

nội tiếp 0180NKM MKC MBC (1)

Mặt khác, vì NM là tiếp tuyến tại M của nửa

đường tròn O nên NMK MBC (2)

Từ (1) và (2) suy ra NKM NMK

Do đó tam giác MNK cân tại N .

3. Ta có IC là trung trực của đoạn AO nên IA IO OA hay tam giác OAI đều cạnh R

Mà IC là đường cao của tam giác đều OAI3 3

2 2

RIC OA

Do K là trung điểm đoạn CI nên 1 3

2 4

RCK CI

Ta có AKC NKM MBC DBC và 090ACK DCB . Suy ra ACK DCB ∽

3.

. 2 2 33

4

R RCA CK CA CB

CD RCD CB CK R

. Vậy 21 1. .2 . 3 3

2 2ABDS AB CD R R R

4. Gọi E là điểm đối xứng với B qua C . Khi đó điểm E cố định và tam giác DBE cân tại

D

Ta có 0 0 0180 180 180DEB DBE MKC DKA DEB DKA

Suy ra tứ giác AKDE nội tiếp

Do đó tâm đường tròn ngoại tiếp tam giác ADK luôn thuộc đường trung trực của đoạn AE cố

định.

Ví dụ 5. Cho tam giác nhọn ABC (với AB AC ) nội tiếp đường tròn O . Gọi D là trung

điểm của cạnh BC , E là hình chiếu của điểm A trên cạnh BC và H là trực tâm của tam

giác ABC . Đường thẳng AD cắt đường tròn O tại điểm thứ hai F .

1) Chứng minh 2 4. .BC DA DF .

x

N

D

MI

C BOA

K

E

Page 45: SỞ GIÁO DỤC VÀ ĐÀO TẠO NAM ĐỊNH

45

2) Tia DH cắt đường tròn O tại điểm G . Chứng minh bốn điểm , , ,A G E D cùng thuộc

một đường tròn.

3) Đường thẳng FE cắt đường tròn O tại điểm thứ hai K . Chứng minh đường thẳng BC

tiếp xúc với đường tròn ngoại tiếp tam giác .GKE

Hướng dẫn:

Xét ADC và BDF có ADC BDF (đối đỉnh)

DAC DBF ( 2 góc nội tiếp chắn cung CF )

nên ADC đồng dạng .BDF g g suy ra AD CD

BD FD

2

. .4

BCAD DF BD DC 2 4. .BC DA DF

Gọi I là điểm đối xứng với A qua O , suy ra AI là đường

kính của O

090ABI ACI ( góc nội tiếp chắn nửa đường tròn)

/ /IB CH (cùng vuông góc với AB ) và / /IC BH (cùng vuông

góc với AC )

IBHC là hình bình hành, suy ra HI đi qua trung điểm D

của BC

, , ,G H D I thẳng hàng, suy ra 090DGA ( góc nội tiếp chắn nửa đường tròn)

Lại có 090DEA (vì E là hình chiếu của A trên BC ), suy ra AGED nội tiếp

4 điểm , , ,A G E D cùng nằm trên một đường tròn.

Vì AGED là tứ giác nội tiếp nên 0 090 90EGD EAD EDA DEF DFE

090 (1)KEB DEF EGD DFE

Vì AGKF là tứ giác nội tiếp nên

0 0 0180 180 90DFE AGK AGD DGE EGK DGE EGK (vì 090AGD )

090EGK EGD DFE (2)

Từ (1) và (2) suy ra KEB EGK (3)

Gọi Et là tia tiếp tuyến của đường tròn ngoại tiếp GKE ( Et và G nằm khác phía đối với EK

), suy ra EGK KEt (góc nội tiếp và góc tạo bởi tia tiếp tuyến và dây cung cùng chắn một

cung) (4)

Từ (3) và (4) suy ra KEB KEt EB Et

Vậy BC là tiếp tuyến của đường tròn ngoại tiếp GKE

Ví dụ 6. Từ một điểm A bên ngoài đường tròn ,O R dựng các tiếp tuyến ,AB AC và cát

tuyến ADE với đường tròn O ( ,B C là các tiếp điểm, ,D E O và D nằm giữa ,A E ).

Đường thẳng qua D vuông góc với OB cắt ,BC BE lần lượt tại ,H K . Gọi I là hình chiếu

của O trên AE .

D

A

B C

G

K H

E

O

I F

Page 46: SỞ GIÁO DỤC VÀ ĐÀO TẠO NAM ĐỊNH

46

1. Chứng minh rằng tứ giác OIBC nội tiếp được một đường tròn.

2. Chứng minh rằng IA là tia phân giác của góc BIC

3. Gọi S là giao điểm của AD và BC . Chứng minh rằng 2.AD AE AC và

2 1 1

AS AD AE .

4. Chứng minh rằng tứ giác IHDC nội tiếp và .DH HK

Hướng dẫn:

1. Vì I là hình chiếu của O trên AE nên suy

ra OI AE

Vì AB là tiếp tuyến tại B của O nên

BO AB

Do đó 090OIA OBA

Vậy tứ giác OIBC nội tiếp được một đường

tròn.

2. Ta có AC là tiếp tuyến tại C của O nên

OC AC

Suy ra 090OCA hay C thuộc đường tròn đường kính OA

Các điểm , , , ,O I A B C cùng thuộc đường tròn đường kính OA và AB AC (tính chất tiếp

tuyến)

Nên suy ra BIA CIA hay IA là tia phân giác của góc BIC

3. Vì AC là tiếp tuyến tại C của O nên ACD AEC

Do đó 2.AC AD

ACD AEC AD AE ACAE AC

*) Các điểm , , , ,O I A B C cùng thuộc đường tròn đường kính OA và AB AC nên

AIC ACS

Suy ra 2.AC AS

ACS AIC AI AS ACAI AC

*) Ta có OI DE nên I là trung điểm của đoạn DE

Do đó 2

1 1 2 2

. .

AD AE AI ID AI ID AI

AD AE AD AE AC AI AS AS

4. Ta có ||DK AB (cùng vuông góc với AB ) nên BAI KDI

Ma tứ giác ABIC nội tiếp nên BAI BCI

Do đó HDI KDI BCI HCI . Vậy tứ giác IHDC nội tiếp.

*) Do các tứ giác IHDC và BDCE nội tiếp nên ||CHI CDI CDE CBE IH BE

Mà I là trung điểm của đoạn DE nên H là trung điểm của DK hay HD HK

Ví dụ 7. Cho đường tròn O dây cung AB khác đường kính. Điểm M trên cung lớn AB

sao cho tam giác MAB luôn là tam giác nhọn; H là hình chiếu của M trên ,AB E và F lần

lượt là hình chiếu của H trên , .MA MB

1. Chứng minh rằng .OM EF

K

H

ISD

A O

B

C

E

Page 47: SỞ GIÁO DỤC VÀ ĐÀO TẠO NAM ĐỊNH

47

2. Trên đoạn MH lấy điểm .I Các tia ,AI BI cắt ,MB MA lần lượt tại , .C D Đường

thẳng qua I song song với AB cắt ,HC HD lần lượt tại ,P Q . Chứng minh rằng .IP IQ

3. Gọi N là giao điểm của OM và .AB Chứng minh rằng 2

2. .

MA HA NA

MB HB NB

Hướng dẫn:

1. Kẻ tiếp tuyến Mx với đường tròn O

Suy ra xMA MBA (1)

Mặt khác 2. .ME MA MF MB MH (hệ thức lượng trong tam giác vuông)

Do đó MEF MBA MEF MBA ∽ (2) Từ (1) và (2) suy ra ||xMA MEF Mx EF

Mà Mx OM nên .OM EF 2. Gọi ', 'A B lần lượt là giao điểm của

PQ với ,MA MB

Vì ' ' ||A B AB nên theo định lts Ta-let ta

có ' '

; ;' '

IP HA IA HA IB AB

IA AB IB HB IQ HA

Suy ra ' '

. . 1' '

IP IA IBIP IQ

IA IB IQ

3. Gọi ,J K lần lượt là hình chiếu của N lên ,MA MB

+) Ta có . .

;. .

MAN MAH

MBN MBH

SNA NJ MA HA S HE MA

NB S NK MB HB S HF MB

Suy ra 2 2

2 2. . . . .

HA NA MA NJ HE MA HE NJ

HB NB MB NK HF MB NK HF (3)

+) Mặt khác, do

0 090 90MEF MBA MFE MAB EMH MAB MFE NMK ∽

Suy ra HE MH

MEH MKNNK MN

∽ (4)

+) Lại có EMH NMK AMN EMH HMN NMK HMN HMB

Suy ra NJ MN

MFH MJNHF MH

∽ (5)

Từ (4) và (5) suy ra . 1HE NJ

NK HF (6). Kết hợp (3) và (6) suy ra

2

2. .

MA HA NA

MB HB NB

Ví dụ 8. (Đề thi tuyển sinh vào trường THPT chuyên Lê Hồng Phong năm học 2016 -

2017 môn Toán chung dành cho các lớp ban KHXH)

Cho tam giác ABC vuông cân tại C nội tiếp đường tròn (O ) . Gọi I là trung điểm

của AC , đường thẳng BI cắt đường tròn (O ) tại điểm thứ hai là K . Đường thẳng AK cắt

đường thẳng BC tại M . 1) Chứng minh: Tứ giác MKIC là tứ giác nội tiếp và MC IO.

x

KJ

B'A'Q

P

DI

N

F

E

H

O

M

A B

C

Page 48: SỞ GIÁO DỤC VÀ ĐÀO TẠO NAM ĐỊNH

48

2) Kẻ KE vuông góc với AC tại E . Tính tỉ số EK

.EI

3) Gọi H là điểm thuộc đường tròn (O ) sao cho tam giác AKH cân tại K . Đường thẳng

AH cắt các đường thẳng KB và KE lần lượt tại F và N . Đường thẳng KE cắt đường

tròn (O ) tại điểm thứ hai là D . Chứng minh KN.KD KF.KB.

Hướng dẫn:

1. +) Trong đường tròn (O ) có 090AKB (góc nội tiếp

chắn nửa đường tròn) 090MKI .

Mặt khác ABC vuông tại C

0 090 90ACB MCI .

Do đó 0180MKI MCI tứ giác MKIC là tứ giác nội

tiếp.

+) Ta có: MAB có AC, BK là các đường cao nên

MI AB ; ABC vuông cân tại C có CO là trung tuyến

nên CO AB.

Do đó MI song song 1CO. ( )

Ta có: AOC cân tại O có OI là đường trung tuyến nên

OI AC; ABC vuông tại C nên BC AC.

Do đó MC song song 2OI. ( )

Từ 1( )và 2( ) suy ra: MCOI là hình bình hành MC IO.

2) Ta có: AOC vuông tại O có OI là đường trung tuyến nên IC IO;

ABC có OI là đường trung bình nên 1

2IO CB. Do đó

1

2IC CB hay 2

BC.

IC

Xét EIK và CIB có 090KEI BCI và KIE BIC (hai góc đối đỉnh).

Do đó EIK đồng dạng CIB 2EK CB

.EI CI

3) Ta có AKH cân tại H AK KH AK KH .

Trong đường tròn (O ) ta có: 1 1 1

2 2 2HFB HB AK HB HK BK; s® s® s® s® s® s®

1

2KDB KB.s® s® Do đó HFB KDB.

Lại có 0180HFB NFB . Suy ra 0180NDB NFB tứ giác BDNF là tứ giác nội tiếp.

Trong đường tròn ngoại tiếp tứ giác BDNF ta có KDF KBN.

Xét KDF và KBN có KDF KBN và BKD chung.

Do đó KDF đồng dạng KBN KD KF

KN.KD KF.KB.KB KN

Ví dụ 9. Cho hai đường tròn O và 'O cắt nhau tại A và B ( O và O'nằm về hai phía của

AB ). Dây cung AD của đường tròn 'O tiếp xúc với đường tròn O tại A , dây cung BC

của đường tròn O tiếp xúc với đường tròn 'O tại B .

D

N

F

H

E

M

K

I

C

BO

A

Page 49: SỞ GIÁO DỤC VÀ ĐÀO TẠO NAM ĐỊNH

49

1. Chứng minh 2

AC BC

BD AD

.

2. Tiếp tuyến chung ngoài về phía A của hai đường tròn tiếp xúc với O tại E , tiếp xúc

với 'O tại F. Đường thẳng EA cắt DF tại I . Chứng minh bốn điểm , , , B I F E cùng

thuộc một đường tròn.

3. Chứng minh tam giác AIF cân và ABI AIF .

Hướng dẫn:

1. Từ giả thiết có ACB BAD

Kết hợp với ABC ADB suy ra

AB AC BCABC BDA

BD AB AD ∽

2 2

. .AB AC BC AC BC

BD AB AD BD AD

2. Ta có AEB BAD và BFD BAD .

Suy ra BFD BEA hay BFI BEI

Vậy bốn điểm , , , B I F E cùng thuộc

một đường tròn.

3. Ta có EIF EBF hay AIF EBF

Ta có ; ABE FEA ABF AFE ABE ABF AEF AFE

EBF AEF AFE

Xét tam giác AEF có FAI AEF AFE FAI AIF AIF cân tại F .

Ta có FEI IBF mà FEI FEA ABE , do đó EBA FBI EBF ABI

Lại có AIF EBF suy ra ABI AIF (đpcm).

Ví dụ 10. Từ điểm M nằm ngoài đường tròn (O ; R) kẻ tiếp tuyến MA (A là tiếp điểm) và cát

tuyến MBC không đi qua tâm O (điểm B nằm giữa M và C). Gọi H là trung điểm của BC.

Đường thẳng OH cắt đường tròn (O ; R) tại hai điểm N, K (trong đó điểm K thuộc cung

BAC). Gọi D là giao điểm của AN và BC.

1. Chứng minh: Tứ giác AKHD là tứ giác nội tiếp.

2. Chứng minh: NAB NBD và BDN ABN .

3. Chứng minh rằng khi đường tròn (O ; R) cố định, điểm M cố định và cát tuyến MBC

thay đổi thì điểm D nằm trên một đường tròn cố định.

Hướng dẫn:

1. Dây BC không đi qua tâm O và H là trung

điểm BC 090OK BC DHK (1)

NK là đường kính của (O ; R) 090NAK

(2)

I

F

D

C

B

A

O O'

E

D

N

K

HB

M

O

A

C

Page 50: SỞ GIÁO DỤC VÀ ĐÀO TẠO NAM ĐỊNH

50

Từ (1) và (2) 4 điểm A, K, H, D thuộc đường tròn đường kính DK Tứ giác AKHD là

tứ giác nội tiếp.

2. H là trung điểm của BC N là điểm chính giữa cung BNC sđBN sđCN

Các góc NAB và NBD là góc nội tiếp lần lượt chắn cung NB và NC của (O ; R)

NAB NBD

Góc BDN có đỉnh nằm trong (O ; R) 1

2BDN ( sđBN sđAKC )=

1

2sđACN

mà góc ABN nội tiếp chắn cung CAN ABN = 1

2sđACN BDN ABN

3. MA là tiếp tuyến của (O ; R) 1

2MAN sđ

1

2ABN (sđAB sđBN ) (2)

và 1

2MDA (sđAB sđCN ) (3).

Mà sđBN sđCN nên từ (2) và (3) MAN MDA MAD cân ở M MA MD .

Do đường tròn (O) cố định và điểm M cố định, nên MA không đổi MD không đổi.

Vậy D thuộc đường tròn ;M MA cố định .

C. Một số bài tập tự luyện.

Bài 1. Cho hai đường tròn , , ',O R O R cắt nhau tại hai điểm A và B . Đường thẳng d

qua B , không vuông góc với AB cắt hai đường tròn , , ',O R O R lần lượt tại các điểm

thứ hai là E và F .

1. Chứng minh rằng .AE AF

2. Đường thẳng 'd qua B , vuông góc với AB cắt hai đường tròn , , ',O R O R

lần lượt tại các điểm thứ hai là C và D . Gọi P là giao điểm của CE và FD . Chứng minh

rằng tứ giác AEPF nội tiếp.

3. Gọi I là trung điểm của .EF Chứng minh rằng tam giác EPF cân và ba điểm

, ,I A P thẳng hàng.

4. Chứng minh rằng khi đường thẳng d quay quanh B thì các điểm ,I P chuyển

động trên các đường tròn cố định.

Bài 2. Từ một điểm A bên ngoài đường tròn O vẽ hai tiếp tuyến ,AB AC và cát tuyến

ADE với đường tròn O ( ,B C là các tiếp điểm, ,D E thuộc đường tròn O và D nằm

giữa ,A E ).

1. Chứng minh rằng 2 . .AB AD AE

2. Đường thẳng AO cắt BC tại H . Chứng minh rằng bốn điểm , , ,D H O E cùng

thuộc một đường tròn.

3. Từ D vẽ đường thẳng song song với BC cắt đường tròn O tại điểm thứ hai là

.K Chứng minh rằng , ,K H E thẳng hàng.

4. Từ D vẽ đường thẳng song song với BE cắt AB và BC lần lượt tại , .F G

Chứng minh rằng D là trung điểm của đoạn .FG

Page 51: SỞ GIÁO DỤC VÀ ĐÀO TẠO NAM ĐỊNH

51

Bài 3. Cho tam giác ABC nhọn nội tiếp đường tròn tâm O. Gọi I là tâm đường tròn nội tiếp

tam giác ABC, đường thẳng AI cắt BC tại D và cắt đường tròn tâm O tại E, E A . Gọi F là

điểm đối xứng với I qua E.

1. Chứng minh BICF là tứ giác nội tiếp.

2. Chứng minh DI.DF=DA.DE.

3. Gọi M là trung điểm của AB, P và Q là hai điểm phân biệt thuộc đường tròn tâm O

sao cho MA là phân giác của góc PMQ và MP không vuông góc với AB. Đường thẳng qua

P và song song với AB cắt đường tròn tâm O tại R, R P . Chứng minh M, R, Q thẳng hàng.

Bài 4. Cho tam giác ABC có 030BAC và nội tiếp đường tròn O bán kính .R Đường

kính PQ của O vuông góc với BC tại M ( P cùng phía với A so với đường thẳng BC

). Gọi ,I J theo thứ tự là chân đường vuông góc hạ từ điểm P xuống AB và .AC

1. Chứng minh các tứ giác AIPJ và CMJP là các tứ giác nội tiếp.

2. Chứng minh ba điểm ,M I và J thẳng hàng.

3. Chứng minh đẳng thức 2

3..

AB AC R

AC AB AC AB

Chuyên đề 6

BẤT ĐẲNG THỨC

A. Lý thuyết cơ bản + 0A B A B + 2 0A với mọi A

+ 2 .x y x y với ,x y là các số thực không âm (BĐT Côsi với 2 số)

B. Luyện tập I. Một số bài toán mở đầu về bất đẳng thức Bài toán 1.1 Cho ,x y là hai số thực, chứng minh 2 2 2 .x y x y (1.1)

Hướng dẫn BĐT (1.1) tương đương với

2 2 2 .x y x y 2

0x y (luôn đúng)

BĐT (1.1) được chứng minh. Bài toán 1.2 Cho , ,x y z là các số thực, chứng minh 2 2 2 . . .x y z x y y z z x (1.2)

Hướng dẫn Cách 1: Áp dụng BĐT (1.1) ta được 2 2 2 .x y x y ; 2 2 2 .y z y z ; 2 2 2 .z x z x

Suy ra 2 2 22 2 . . .x y z x y y z z x 2 2 2 . . .x y z x y y z z x

BĐT (1.2) được chứng minh.

Cách 2: Bất đẳng thức (1.2) tương đương với 2 2 22 2 . . .x y z x y y z z x

2 2 2 2 2 22 2 2 0x y xy y z yz z x zx

2 2 2

0x y y z z x (luôn đúng)

BĐT (1.2) được chứng minh.

Page 52: SỞ GIÁO DỤC VÀ ĐÀO TẠO NAM ĐỊNH

52

Bài toán 1.3

Cho , ,x y z là các số thực không âm, chứng minh 33x y z xyz (1.3)

Hướng dẫn Cách 1 Áp dụng BĐT Côsi với hai số ta được

2x y xy ; 3 32 .z xyz z xyz ; 3 32 .x y z xyz xy z xyz

Mặt khác, áp dụng BĐT Côsi với hai số ta được:

3 3. 2 . .xy z xyz xy z xyz

3 3. 2xy z xyz xyz

Do đó ta suy ra 3 34x y z xyz xyz

33x y z xyz

BĐT (1.3) được chứng minh. Cách 2

BĐT (1.3) tương đương với 3 3 3

3 33 33 33x y z x y z

Sử dụng đẳng thức 3 3 3 2 2 23a b c abc a b c a b c ab bc ca và

2 2 22 2 2 1 1 1

02 2 2

a b c ab bc ca a b b c c a

BĐT (1.3) được chứng minh. Bài toán 1.4

Cho ,x y là hai số thực dương, chứng minh 1 1 4

x y x y

(1.4)

Hướng dẫn Cách 1

BĐT (1.4) tương đương với 1 1

4x yx y

2 4x y

y x 2

x y

y x (luôn đúng theo BĐT Côsi với 2 số)

BĐT (1.4) được chứng minh. Cách 2 Áp dụng BĐT Côsi với hai số ta được

2x y xy ; 1 1 1 1

2. .x y x y

Suy ra 1 1

4x yx y

BĐT (1.4) được chứng minh. Nhận xét: Từ BĐT (1.4) ta có thể mở rộng được các BĐT sau:

+ Cho , ,x y z là các số thực dương, chứng minh 1 1 1 9

.x y z x y z

+ Cho n số thực dương, chứng minh 2

1 2 1 2

1 1 1

n n

n... .

x x x x x ... x

II. Một số bài toán vận dụng Bài toán 2.1

Page 53: SỞ GIÁO DỤC VÀ ĐÀO TẠO NAM ĐỊNH

53

Cho ,a b là các số thực, chứng minh 2 2 1a b ab a b (2.1)

Hướng dẫn Theo bài toán (1.2) ta được 2 2 2 . . .a b c a b b c c a Chọn 1c ta được 2 2 1a b ab a b BĐT (2.1) được chứng minh. Nhận xét: Từ bài toán (2.1) ta có thể phát biểu bài toán sau: “Cho ,a b là các số thực dương

thỏa mãn 1ab , chứng minh 2 2 .a b a b (Bỏ điều kiện “ ,a b là các số thực dương” thì

BĐT vẫn đúng). Cách 1: Áp dụng trực tiếp bài toán (2.1) với 1ab ta suy ra 2 2 .a b a b Cách 2

Do 1

1ab ab

BĐT cần chứng minh trở thành 2

2

1 1a a

a a

21 1

2 0a aa a

1 12 1 0.a a

a a

Áp dụng BĐT Côsi với 2 số ta được

12a

a

Suy ra 1 1

2 1 0.a aa a

(luôn đúng)

Bất đẳng thức 2 2a b a b được chứng minh. Bài toán 2.2 Cho , ,a b c là các số thực dương

Chứng minh 2 2 2 3 2a b c a b c (2.2)

Hướng dẫn Theo bài toán (1.1) ta được

2 2 21 2 ; 1 2 ; 1 2a a b b c c

Suy ra 2 2 2 3 2a b c a b c

Bất đẳng thức (2.2) được chứng minh. Nhận xét: Từ bài toán (2.2) ta có thể phát biểu bài toán: “ Cho , ,a b c là các số thực dương,

chứng minh 3.a b c Chứng minh 2 2 2 3a b c và 2 2 2 .a b c a b c Hướng dẫn

Từ BĐT (2.2) ta được 2 2 2 3 2a b c a b c , kết hợp với 3a b c ta suy ra các

BĐT đã cho được chứng minh. Bài toán 2.3 Cho , ,a b c là các số dương, chứng minh

4 4 4 1 1 1

2 2 2a b c b c a c a b a b c

(2.3)

Hướng dẫn

Áp dụng kết quả bài toán 4.1 ta được 1 1 4 1 1 4

;2a b a b a b a c c a b

Từ đó BĐT (2.3) được chứng minh. Nhận xét: Từ bài toán 2.3 ta có thể phát biểu bài toán: “Cho , ,a b c là các số dương và

1.a b c Chứng minh 4 4 4 1 1 1

.1 1 1a b c a b c

Hướng dẫn Áp dụng trực tiếp bài toán 2.3 khi thay 1.a b c

Page 54: SỞ GIÁO DỤC VÀ ĐÀO TẠO NAM ĐỊNH

54

Bài toán 2.4

Cho , ,a b c là các số dương thỏa mãn 3

2a b c

Chứng minh 1 1 1 15

.2

a b ca b c

(2.4)

Hướng dẫn Áp dụng BĐT Côsi với hai số ta được

1 1 14 4; 4 4; 4 4;a b c

a b c

Suy ra 1 1 1

12 3a b c a b ca b c

Do 3

2a b c nên ta được

1 1 1 15.

2a b c

a b c

Bất đẳng thức (2.4) được chứng minh. Nhận xét:

+ Xuất phát từ dấu bằng xảy ra khi 1

2a b c và khi

1

2a thì

14a

a nên ta áp dụng

BĐT Côsi với 2 số là 4a và 1

a

+ Bài toán tương tự xuất phát từ bài toán 2.4: “Cho , ,a b c là các số dương thỏa mãn

3

2a b c . Chứng minh

2 2 2

1 1 1 27

2a b c

a b c

Hướng dẫn: Áp dụng BĐT Côsi ta được 3

2 2

1 18 8 3 64 16 12.a a a

a a

II. Một số bài toán luyện tập Bài 1 Cho , ,a b c là các số thực không lớn hơn 1 và 2a b c

Chứng minh 1.ab bc ca (1) Hướng dẫn Do , ,a b c là các số thực không lớn hơn 1 nên 1 ;1 ;1a b c là các số không âm.

1 1 0a b 1 ab a b

Tương tự ta đươc: 1 ; 1bc b c ca c a

3 2ab bc ca a b c

Do 1a b c 1ab bc ca Bất đẳng thức (1) được chứng minh. Bài 2 Cho , ,a b c là các số thực dương không lớn hơn 1 và 2a b c

Chứng minh 2.ab bc ca (2) Hướng dẫn Vì 0 1;0 1;0 1;a b c nên ta có

( 1) 0ab a a b ; ( 1) 0bc b b c ; (a 1) 0ca c c

ab bc ca a b c 2ab bc ca

Bất đẳng thức (2) được chứng minh. Bài 3

Cho , ,a b c là các số thực dương, chứng minh ab bc ca

a b cc a b (3)

Page 55: SỞ GIÁO DỤC VÀ ĐÀO TẠO NAM ĐỊNH

55

Hướng dẫn

BĐT (3) tương đương với 2. 2ab bc ca

a b cc a b

2 2 2ab bc bc ca ab ca

a b cc a a b c b

. . . 2 2 2a c b a b c

b c a a b cc a a b c b

Do 2; 2; 2;c a b a c b

a c a b b c

Bất đẳng thức (3) được chứng minh. Bài 4

Cho , ,a b c là các số thực dương, chứng minh 2 2 2a b c

a b cb c a

(4)

Hướng dẫn Áp dụng BĐT Côsi với hai số ta được

2

2a

b ab

; 2

2b

c bc ;

2

2c

a ca

2 2 2

2a b c

b c a a b cb c a

2 2 2a b ca b c

b c a

Bất đẳng thức (4) được chứng minh. Bài 5

Cho ,a b là các số thực dương, chứng minh 2 2

.a b

a bb a

(5)

Hướng dẫn Cách 1 Áp dụng BĐT Côsi ta được

2

2a

b ab

; 2

2b

a ba

2 2

2a b

b a a bb a

2 2

.a b

a bb a

Bất đẳng thức (5) được chứng minh. Bài 6

Cho , ,a b c là các số thực dương, chứng minh 3 3 3

2 2 2.a b c

a b cb c a (6)

Hướng dẫn Áp dụng BĐT Côsi ta được:

3 3 32 2 22 ; 2 ; 2

a b cab a bc b ca c

b c a

3 3 3

2 2 22a b c

ab bc ca a b cb c a

3 3 3

2 2 2 2 2 2 .a b c

a b c a b c ab bc cab c a

Theo BĐT (1.2) ta được 2 2 2 0a b c ab bc ca 3 3 3

2 2 2.a b c

a b cb c a

Page 56: SỞ GIÁO DỤC VÀ ĐÀO TẠO NAM ĐỊNH

56

Bất đẳng thức (6) được chứng minh. Bài 7

Cho , ,a b c là các số thực dương, chứng minh 3 3 3

2 2 2.

a b ca b c

b c a (7)

Hướng dẫn Áp dụng BĐT (1.3) ( BĐT Côsi với ba số) ta được

3 3 3

2 2 23 ; 3 ; 3 ;

a b cb b a c c b a a c

b c a

3 3 3

2 2 2.

a b ca b c

b c a

Bất đẳng thức (7) được chứng minh. Bài 8

Cho số thực 1x , chứng minh 2 1x x (8) Hướng dẫn

BĐT (8) tương đương với 2 1 0x x

1 1 2 1 0x x

2

1 1 0x (luôn đúng)

Bất đẳng thức (8) được chứng minh. Bài 9 Cho , ,a b c là các số thực thỏa mãn 1; 2; 3a b c và 9a b c

Chứng minh 1 2 3 3a b c (9) Hướng dẫn Áp dụng BĐT Côsi với hai số ta được

1 1 2 1a a ; 2 1 2 2b b ; 3 1 2 1c c

3 2 1 2 3a b c a b c

Do 9a b c nên 1 2 3 3a b c Bất đẳng thức (9) được chứng minh. Bài 10

Cho số thực x không âm, chứng minh 21 2 2x x x (10) Hướng dẫn

Bất đẳng thức (10) tương đương với 1 2 1x x x x

1 2 ( 1) 0x x x x

2

1 0x x (luôn đúng, vì 1x x )

Bất đẳng thức (10) được chứng minh. Bài 11 Cho , ,a b c là các số thực duong thỏa mãn 1a b c

Chứng minh 2 2 2 5

2a a b b c c (11)

Hướng dẫn Áp dụng BĐT (10) ta được

21 2 2a a a ; 21 2 2b b b ; 21 2 2c c c ;

2 2 23 2 2a b c a a b b c c

Page 57: SỞ GIÁO DỤC VÀ ĐÀO TẠO NAM ĐỊNH

57

2 2 2 5

2a a b b c c (Vì 1a b c )

Bất đẳng thức (11) được chứng minh.

PHẦN II - CÁC ĐỀ LUYỆN THI VÀO LỚP 10

ĐỀ SỐ 01 Thời gian làm bài: 120 phút

Bài 1. (2,5 điểm)

1) Tìm điều kiện xác định của biểu thức 1

1x .

2) Đồ thị hàm số 2y x cắt trục Ox tại điểm M. Tìm tọa độ điểm M.

3) Hình tròn có độ dài đường kính là 10. Tính diện tích hình tròn đó ?

4) Cho hàm số 1 2 4 3f x x . Hàm số đã cho đồng biến hay nghịch biến? Vì

sao?

5) Tìm các giá trị của m để đường thẳng 22 1 2y m x m vuông góc với đường

thẳng ?y x

Bài 2: (2,5 điểm) 1. Cho phương trình 2 22 1 0x mx m m (1) ( m là tham số).

a) Tìm các giá trị của m để phương trình (1) có 2 nghiệm phân biệt.

b) Gọi 1 2,x x là hai nghiệm phân biệt của phương trình (1) . Tìm các giá trị của m sao

cho

2 21 2 1 21 1 2x x x x .

2. Giải hệ phương trình

2 2

2

1

2 1 2 3 4

x y y

x x y y

Bài 3: (3,0 điểm) Cho đường tròn O và đường thẳng d cố định, đường thẳng d không cắt đường

tròn O . Một điểm M di động trên đường thẳng d . Vẽ hai tiếp tuyến ,MA MB phân biệt

và cát tuyến MCD với đường tròn O (với ,A B là các tiếp điểm, C nằm giữa ,M D và

CD không đi qua O ). Gọi N là điểm thuộc đường tròn O sao cho DN song song với

AB , I là giao điểm của CN và AB . Chứng minh rằng:

1. IC BC

IA BD

2. IA IB 3. Điểm I chạy trên một đường cố định khi M di động trên đường thẳng d .

Bài 4. (1,0 điểm) Giải phương trình

3 2 38 19 12 2 3 5 ( 2)x x x x x x x .

Bài 5: (1,0 điểm)Cho , ,a b c là các số thức dương thỏa mãn 22 232 12a b c b c .

Page 58: SỞ GIÁO DỤC VÀ ĐÀO TẠO NAM ĐỊNH

58

Chứng minh rằng 3 3 3 3

43 3 3

a b c

b bc c ca a ca

ĐỀ SỐ 02

Thời gian làm bài: 120 phút Phần I. Trắc nghiệm (2 điểm).

Hãy chọn phương án trả lời đúng và viết chữ cái đứng trước phương án đó vào bài làm.

Câu 1. Rút gọn biểu thức 3 2 2 2 được kết quả bằng:

A. 3 2 . B. 0. C.

3 3 2 . D.

3 2 .

Câu 2. Tập nghiệm của phương trình 2x là:

A. 4 . B. 4 . C. . D.

4; 4 .

Câu 3. Hàm số y = f(x) = ax + 2019 có tính chất nào sau đây khi biết f(1) > f(1) ?

A. Đồng biến trên R. B. Đồng biến khi x > 0.

C. Đồng biến khi x < 0. D. Nghịch biến trên R.

Câu 4. Đường thẳng y = 3x + 1 tạo với trục Ox một góc . Đường thẳng nào sau đây tạo với

trục Ox một góc lớn hơn ?

A. y = x + 1. B. y = 4x +1. C. y = 3x + 2. D.

y = 4x + 1.

Câu 5. Số nghiệm của phương trình x4 + x2 – 2020 = 0 là

A. 1. B. 2. C. 3. D.

4.

Câu 6. Cho biết góc nhọn và sin = 2.cos . Khi đó ta có:

A. cot = 2. B. cot =0,5. C. cot = 1. D.

cot =0,5.

Câu 7. Tam ABC vuông tại A, đường cao AH. Biết AB = 3 cm; AC = 4 cm. Độ dài đoạn BH bằng:

A. 2 cm. B. 3,2 cm. C. 1,8 cm. D.

5 cm. Câu 8. Một hình nón có đường sinh 5 cm, bán kính đáy 3 cm. Thể tích của hình nón đó bằng:

A. 312 mc . B. 315 mc . C. 316 mc . D.336 mc .

Phần II – Tự luận (8 điểm) Câu 1. (1,5 điểm) Rút gọn các biểu thức sau:

a) 3 13 6

2 3 4 3 3

.

b) x y y x x y

xy x y

với x > 0 ; y > 0 ; x y .

Câu 2. (1,5 điểm) Trong mặt phẳng tọa độ Oxy, cho đường thẳng (d): y k 1 x 3 và parabol

(P): 2y x .

a) Chứng minh rằng với mọi giá trị của k thì đường thẳng (d) luôn cắt parabol (P) tại hai điểm phân biệt;

Page 59: SỞ GIÁO DỤC VÀ ĐÀO TẠO NAM ĐỊNH

59

b) Gọi y1; y2 là tung độ các giao điểm của đường thẳng (d) và parabol (P). Tìm k sao cho:

1 2 1 2y y y y .

Câu 3. (1,0 điểm) Giải hệ phương trình:

2 1 2 1

2 2 1

x y

x y

.

Câu 4. (3,0 điểm) Cho đường tròn (O;R) đường kính AB. Trên tia đối của tia BA lấy điểm C. Qua

C vẽ đường thẳng d vuông góc với AC. M là một điểm trên d (M không trùng với C). MA cắt

đường tròn tại điểm thứ hai là P; MB cắt đường tròn tại điểm thứ hai là Q. Qua P vẽ tiếp tuyến

của đường tròn (O;R) cắt d tại điểm K.

a) Chứng minh bốn đểm P, B, C, M cùng thuộc một đường tròn.

b) Chứng minh tam giác KPM cân tại K.

c) Chứng minh KQ là tiếp tuyến của đường tròn (O;R).

Câu 5. (1,0 điểm) Giải phương trình 2 22 3 2 2 2x x x x x .

ĐỀ SỐ 03

Thời gian làm bài: 120 phút

Câu 1. (2,0 điểm)

6) Tìm điều kiện xác định của biểu thức 1

1x.

7) Đồ thị hàm số 2 2 y x cắt trục Oy tại điểm M. Tìm tọa độ điểm M.

8) Hình vuông ABCD có độ dài cạnh là 5 nội tiếp trong đường tròn ( ; )O R . Tính bán

kính đường tròn ( ; )O R .

9) Cho biểu thức . 1 2 5 4 3 P x x . Tính giá trị của P khi 3x .

Câu 2. (1,5 điểm) Cho biểu thức 2 2

:11 1

x xP

xx x

(với 0x và 1x ).

1) Rút gọn biểu thức P .

2) Tìm x để 5

.2

P

Câu 3. (2,5 điểm)

1) Cho phương trình 2 4 1 0 x x m * ( m là tham số).

a) Tìm m để phương trình * có nghiệm.

b) Giả sử 1 2;x x là hai nghiệm của phương trình * . Tìm m sao cho

3 31 2 1 21 1 28 x x x x .

2) Giải hệ phương trình 21 2 4 4

4 3.

x y y

x y

Câu 4. (3,0 điểm) Từ điểm M nằm ngoài đường tròn (O ; R) kẻ tiếp tuyến MA (A là tiếp

điểm) và cát tuyến MBC không đi qua tâm O (điểm B nằm giữa M và C). Gọi H là trung

điểm của BC. Đường thẳng OH cắt đường tròn (O ; R) tại hai điểm N, K (trong đó điểm

K thuộc cung BAC). Gọi D là giao điểm của AN và BC.

Page 60: SỞ GIÁO DỤC VÀ ĐÀO TẠO NAM ĐỊNH

60

1) Chứng minh: Tứ giác AKHD là tứ giác nội tiếp.

2) Chứng minh: NAB NBD và BDN ABN .

3) Chứng minh rằng khi đường tròn (O ; R) cố định, điểm M cố định và cát tuyến MBC

thay đổi thì điểm D nằm trên một đường tròn cố định.

Câu 5. (1,0 điểm) Cho các số thực dương , ,x y z thỏa mãn 1xy và 1z .

Chứng minh bất đẳng thức

3 2 3.

1 1 3 1 2

x y z

y x xy

ĐỀ SỐ 04

Thời gian làm bài: 120 phút I. TRẮC NGHIỆM ( 2 điểm) Hãy chọn chữ cái đứng trước phương án trả lời đúng và ghi vào bài làm

Câu 1. Biểu thức

2

1

1 x xác định khi và chỉ khi

A. x > 1. B. x < 1. C. x 1 . D. x = 1.

Câu 2. Biểu thức 3 3

3 31 3 1 3 có giá trị bằng

Câu 3. Nếu đồ thị hàm số y = 2x + m cắt trục hoành tại điểm có hoành độ bằng 3 thì m có giá trị bằng

A. 6. B. 3. C. 2. D. 6 .

Câu 4. Hàm số y = 12

m

x2 đồng biến khi x < 0 nếu

A. m > 12

. B. m < 12

. C. m 12

. D. m 12

.

Câu 5. Số nghiệm của phương trình 4 24 3 0x x là:

A. 1 ; B. 2; C. 3; D. 4. Câu 6. Lập phương trình bậc hai ẩn y có các nghiệm là bình phương các nghiệm của phương trình

2 2 0x x , ta được kết quả là:

A. 2 5 4 0y y ; B. 2 2 0y y ; C. 2 2 0y y ; D. 2 3 4 0y y .

Câu 7. Khi đường tròn (O’;R’) đựng đường tròn (O; R) ta có hệ thức: A. OO’ > R+R’; B. OO’ < R’ – R ; C. OO’ > R’ – R; D. OO’ = R’ – R.

Câu 8. Một hình trụ và một hình nón có chiều cao và bán kính đáy bằng nhau. Tỷ số thể tích giữa hình nón và hình trụ là:

A. 1

2 . B.

1

3. C. 2

3.

D. 2.

II. TỰ LUẬN: (8 điểm)

Câu 1. ( 1,5 điểm ) Cho biểu thức A = x 1 2 3

+ x + 2 x + 1 x + 3 x + 2

( với x 0 ).

1) Rút gọn biểu thức A.

2) Chứng minh rằng khi x = 4 2 3 thì biểu thức A có giá trị bằng 3 1 . Câu 2. (1,5 điểm) Cho phương trình x2 – 3(m – 1)x + 2m2 – 6m = 0 (1)

1) Chứng minh phương trình (1) luôn có nghiệm với mọi giá trị của m. 2) Gọi x1, x2 là các nghiệm của phương trình (1). Chứng minh giá trị của biểu thức:

B = 2

1 2 1 2 1 22 6 6 9x x x x x x không phụ thuộc vào giá trị của m.

A. 2. B. 2 3 . C. 32 2 . D. 2 3 .

Page 61: SỞ GIÁO DỤC VÀ ĐÀO TẠO NAM ĐỊNH

61

Câu 3. (1,0 điểm) Cho hệ phương trình: 2

2

( 1)

2

mx y m

x my m

1) Chứng minh rằng hệ phương trình luôn có nghiệm duy nhất . 2) Giả sử (x0 ; y0) là nghiệm của hệ phương trình trên. Khi m thay đổi, chứng minh rằng điểm

M(x0; y0) luôn thuộc một đường thẳng cố định. Câu 4. ( 3 điểm ) Cho đường tròn (O; R) và điểm A nằm ngoài đường tròn (O). Qua A vẽ tiếp tuyến

AB tiếp xúc với đường tròn (O) tại B. Vẽ một đường thẳng qua A cắt đường tròn tại hai điểm M và N (M nằm giữa A và N). Qua M kẻ đường thẳng song song với AB cắt BN tại E. Gọi I là trung điểm của ME. Vẽ dây BQ của đường tròn (O) sao cho BQ đi qua điểm I. a) Chứng minh hai tam giác BMI và BQM đồng dạng. b) Chứng minh tứ giác QIEN nội tiếp. c) Chứng minh BM. QN = BN. MQ.

Câu 5. (1,0 điểm) Giải phương trình 2 2 2x x 1 x x 1 x x 2 .

ĐỀ SỐ 05 Thời gian làm bài: 120 phút

Câu 1 (2,0 điểm).

1) Tìm tập nghiệm của phương trình 2 3 .x x

2) Tìm m để đường thẳng 2 1y x và đường thẳng 2 3y x m cắt nhau tại một

điểm trên trục tung.

3) Lập một phương trình bậc hai ẩn x nhận 2 3 và 2 3 là nghiệm.

4) Cho ( ; )O R và dây 2AB R . Tìm số đo cung lớn .AB

Câu 2 (1,5 điểm). Cho biểu thức 2 2

11

x x x xA

x x x

(với 0x ).

a) Rút gọn .A b) Chứng minh rằng nếu 0 1x thì 0.A A

Câu 3 (2,5 điểm).

1) Cho phương trình 2 3( 1) 3 4 0x m x m (với x là ẩn, m là tham số).

a) Chứng minh rằng với mọi giá trị của m thì phương trình đã cho luôn có nghiệm. b) Tìm các giá trị của tham số m để phương trình có hai nghiệm phân biệt là độ dài

hai đường chéo của một hình thoi có độ dài cạnh là 5

.2

2) Cho hệ phương trình 2

3

2

x my m

mx y m

(với ,x y là ẩn, m là tham số). Tìm m để hệ

phương trình có nghiệm duy nhất ;x y thỏa mãn 2 2 0.x mx y

Câu 4 (3,0 điểm).

Cho tam giác ABC có ba góc nhọn, nội tiếp đường tròn (O) (AB < AC). Các tiếp tuyến với (O) tại B và C cắt nhau tại N. Vẽ dây AM song song với BC. Đường thẳng MN cắt đường tròn (O) tại M và P.

1) Chứng minh rằng ON vuông góc với BC. Tính độ dài đoạn BC (đơn vị cm) biết

2 2

1 1 1.

25OB NC

Page 62: SỞ GIÁO DỤC VÀ ĐÀO TẠO NAM ĐỊNH

62

2) Chứng minh rằng .BP CP

AC AB

3) Chứng minh rằng BC, ON và AP đồng quy. Câu 5 (1,0 điểm).

1) Giải phương trình 2 2 5 3 2 1.x x x x

2) Cho a, b là hai số dương thỏa mãn 1a b . Tìm giá trị nhỏ nhất của biểu thức

2 23 3 2 2 3

.2

M a b a b ab

ĐỀ SỐ 06

Thời gian làm bài: 120 phút Phần I. Trắc nghiệm (2,0 điểm) Hãy chọn phương án trả lời đúng và viết chữ cái đứng trước phương án đó vào bài làm

Câu 1. Căn thức 2

1

( 2)x có nghĩa khi

A. 0x . B. 2x . C. 2x . D. 2x .

Câu 2. Với x , hàm số 21

2y x đồng biến khi

A. 0x . B. 0x . C. 1

2x . D.

1

2x .

Câu 3. Đường thẳng 2y mx m và đường thẳng 2( 1) 2y m x cắt nhau tại một điểm trên trục

tung khi A. 2m . B. 0m . C. 2m . D. 2m .

Câu 4. Đường thẳng 1

2y x k tiếp xúc với parabol

2

2

xy khi

A. 1

2k . B.

1

2k . C.

1

8k . D.

1

8k .

Câu 5. Trong các phương trình sau, phương trình nào có nghiệm dương?

A. 2 2 3 0x x . B. 2 2 3 0x x . C. 2 4 5 0x x . D. 2 3 2 0x x . Câu 6. Cho đường tròn (O, R) và (O, r), biết R+r=OO’. Khi đó, số tiếp tuyến chung của 2 đường tròn là

A. 3. B. 2. C. 1. D. 0. Câu 7. Độ dài cung 600 của một đường tròn có bán kính bằng 2cm là

A. 2

3 cm. B. 4cm. C.

3

2 cm. D.

1

3 cm.

Câu 8. Từ một khối gỗ hình trụ có chiều cao 15cm, người ta tiện thành một khối hình nón có thể tích lớn nhất. Biết phần gỗ bỏ đi có thể tích là 20cm3. Bán kính khối gỗ đó lúc đầu bằng

A. 3cm. B. 2cm. C. 1,5cm. D. 2 . Phần II. Tự luận (8,0 điểm) Bài 1. (1,5 điểm) Rút gọn các biểu thức sau:

a) 3 13 48A b) 2

:a a b b b

B ab a ba b a b

với

0 0,a , b a b .

Bài 2. (1,5 điểm) Cho hai hàm số 2y x và 3y kx với k là tham số.

a) Khi 3k , tìm tọa độ giao điểm của hai đồ thị hai hàm số trên.

Page 63: SỞ GIÁO DỤC VÀ ĐÀO TẠO NAM ĐỊNH

63

b) Chứng minh rằng với mọi giá trị k, đồ thị của hai hàm số đã cho luôn cắt nhau tại hai

điểm phân biệt 1 1;A x y và 2 2;B x y . Tìm tất cả các giá trị của m sao cho 2 21 2 36y y .

Bài 3. (1,0 điểm) Giải hệ phương trình 2 2

1

7

x y

x xy y

Bài 4. (3,0 điểm) Cho đường tròn tâm O và dây cung AB. Lấy điểm E trên dây cung AB (E khác A và B). Qua E vẽ dây cung CD của đường tròn (O). Trên hai tia DA và DB lấy điểm P và Q đối xứng qua E. Gọi I là tâm đường tròn đi qua C và tiếp xúc với PQ tại E, đường tròn (I) cắt AB tại M. Chứng minh rằng: a) . .EA EB EC ED . b) CMA ∽ QED .

c) MA MB . Bài 5. (1,0 điểm)

a) Tìm giá trị nhỏ nhất của biểu thức 9 6 1 2P x x x x với 0x .

b) Trong một cuộc thi bóng đá thể thức vòng tròn một lượt tính điểm, đội thắng được 3 điểm, đội hòa được 1 điểm, đội thua 0 điểm, có 4 đội tham gia. Cuối cùng 4 đội nhận được lần lượt lần lượt 5, 1, x, 6 điểm. Tìm x?

ĐỀ SỐ 07 Thời gian làm bài: 120 phút

Phần I. Trắc nghiệm (2,0 điểm) Hãy viết vào bài làm chữ cái đứng trước phương án trả lới đúng:

Câu 1. Điều kiện xác định của biểu thức 2 x là

A. 2x B. x > 2 C. 2x D. 2x

Câu 2. Tập nghiệm của phương trình 3 0x x là

A. 1;1S B. 0;1S C. 1; 0; 1S D. 1S

Câu 3. Phương trình 2 6 8 0x x có tổng hai nghiệm là A. 6 B. 3 C. 8 D. 6

Câu 4. Đồ thị hàm số 22y x và đồ thị hàm số y = 8x + m (m là tham số), không có điểm chung

khi và chỉ khi A. 8m B. 8m C. 32m D. 32m

Câu 5. Hệ phương trình 2 4

3 4

mx y

x y m

(m là tham số), có một nghiệm duy nhất khi và chỉ khi

A. 2

3m

B.

3

2m

C.

3

2m

D. 4m

Câu 6. Tam giác EGH có độ dài các cạnh là: HE = 6 cm, HG = 8 cm, EG = 10 cm. Khi đó độ dài đường cao HK là A. 4,8 cm B. 6,4 cm C. 48 cm D. 2,4 cm

Câu 7. Cho tứ giác MNPQ nội tiếp đường tròn (O; R), biết 2MN R . Ta có MQN bằng

A. 300 B. 450 C. 600 D. 900

Câu 8. Hình trụ có chiều cao bằng 6 cm, thể tích bằng 2150 cm . Bán kính đáy của hình trụ đó là

A. 25cm B. 20cm C. 5cm D. 50cm Phần II. Tự luận (8,0 điểm) Bài 1. (1,5 điểm)

1) Chứng minh đẳng thức thức: 1 33 2 2019

48 1 675 32 11 3 1 3

2) Rút gọn biểu thức 1 2

11 1

xB

xx x

với 0x và 1x .

Page 64: SỞ GIÁO DỤC VÀ ĐÀO TẠO NAM ĐỊNH

64

Bài 2. (1,5 điểm) Cho phương trình 2 2 3 0x mx m (1) , (m là tham số, x là ẩn).

1) Chứng minh phương trình (1) luôn có hai nghiệm phân biệt 1x , 2x với mọi m.

2) Tìm tất cả các giá trị của m để các nghiệm 1x , 2x thỏa mãn: 21 22 6x mx

Bài 3. (1,0 điểm) Giải hệ phương trình: 2 16

2

x xy y

x y

Bài 4. (3,0 điểm) Cho đường tròn (O) và điểm K nằm bên ngoài đường tròn. Từ K kẻ cát tuyến KAB đi qua O, cát tuyến KCD không đi qua O (KA < KB, KC < KD). Gọi I là giao điểm của AD và BC, H là hình chiếu của I trên AB.

1) Chứng minh tứ giác ACIH nội tiếp đường tròn và AIH ABD . 2) Chứng minh KCH KOD ∽ . 3) Vẽ KM là tiếp tuyến của (O) (M là tiếp điểm). Chứng minh ba điểm I, H, M thẳng hàng. Bài 5. (1,0 điểm) Cho x, y, z là các số dương và x + y + z = 1.

Chứng minh rằng: 2 22 2

11

xy z x y

xy

.

ĐỀ SỐ 08

Thời gian làm bài: 120 phút PHẦN I : Trắc nghiệm ( 2,0 điểm )

Câu 1: Cho hàm số 2

2 1 0( )

3 0

x khi xf x

x khi x

khẳng định nào sau đây sai ?

A. ( 1) 3f . B. (10) 19f . C. ( 1) 2f . D. ( 10) 97f .

Câu 2: Trong mặt phẳng Oxy, biết đồ thị hàm số 2y ax đi qua điểm A( 2019;1).

Giá trị của hàm số tại 2019x bằng

A. 2019 B. a C. 1 D.0

Câu 3: Điều kiện để biểu thức 2

2x x xác định là

A. x B. 0x C. 0x D. 0x Câu 4: Xét m là số nguyên tố. Có bao nhiêu giá trị tham số m để hàm số (5 ) 2019y m x

đồng biến trên . A. 1 B. 2 C.3 D. 4 Câu 5: Phương trình nào sau đây có hai nghiệm trái dấu ? A. 2 3 1 0x x . B. 25 9 2 0x x . C. 25 3 2019 0x x . D. 2 3 0x x . Câu 6: Cho tam giác ABC đều cạnh bằng 2. Diện tích tam giác bằng

A. 2 B. 4 C. 3 D. 2 3 Câu 7: Một khối trụ có diện tích xung quanh và thể tích bằng nhau. Bán kính đáy khối trụ bằng

A. 2 B. 4 C. 1 D. 2

Câu 8: Một hình nón có đường cao bằng bán kính đáy bằng 3 2 . Diện tích xung quanh khối nón bằng

A. 6 2 . B.8 2 . C.18 . D.18 2 . PHẦN II: Tự luận ( 8 điểm )

Câu 1:( 1,5 điểm ) Cho biểu thức 2 1 1

:21 1 1

x x xP

x x x x x

(với 0; 1x x )

1). Rút gọn P. 2). Tìm giá trị lớn nhất của P.

Page 65: SỞ GIÁO DỤC VÀ ĐÀO TẠO NAM ĐỊNH

65

Câu 2:( 1,5 điểm ) Cho phương trình 2 2

2x m

x

1). Tìm m để phương trình có đúng 1 nghiệm. 2). Tìm m để phương trình có hai nghiệm phân biệt 1 2,x x thoả mãn

21 2 1 2( 2) 6x x m x x

Câu 3:( 1,0 điểm ) Tìm một số tự nhiên có hai chữ số nếu chia số đó cho tổng hai chữ số được thương là 6, nếu cộng tích hai chữ số với 25 thì được số có hai chữ số viết theo thứ tự ngược lại với số cần tìm. Câu 4:( 3,0 điểm ) Cho đường tròn (O; R) đường kính AB, bán kính CO vuông góc với AB, M là điểm bất kỳ trên cung nhỏ AC( M khác A và C), BM cắt AC tại H. Gọi K là hình chiếu vuông góc của H trên AB.

1) Chứng minh CBKH là tứ giác nội tiếp và ACM ACK . 2) Trên đoạn thẳng BM lấy điểm E sao cho BE=AM . Chứng minh tam giác ECM là tam giác vuông cân tại C. 3) Gọi d là tiếp tuyến của đường tròn (O) tại A. Cho P là một điểm nằm trên d sao cho hai

điểm P, C nằm trong cùng nửa mặt phẳng bờ AB và .AP MB

RMA

. Chứng minh đường thẳng

PB đi qua trung điểm đoạn thẳng HK.

Câu 5:( 1,0 điểm ) Giải hệ phương trình 2 21 . 1 1

6 2 1 4 6 1

x x y y

x x xy xy x

ĐỀ SỐ 09 Thời gian làm bài: 120 phút

PHẦN I : Trắc nghiệm ( 2,0 điểm )

Câu 1: Điều kiện để biểu thức 1

1x x xác định là

A. 1x . B. 1x . C. 1x . D. 1x và 0x .

Câu 2: Cho hàm số bậc hai 2axf x biết 4f m f m . Giá trị của hàm số tại

x m bằng A.1 B. 2 C. 3 D. 0 Câu 3: Đường thẳng 6y x cắt trục ox, oy lần lượt tại A và B. Diện tích tam giác OAB

bằng A.6 B. 3 C. 18 D. 12

Câu 4: Cho hàm số axf x b , biết 1 1f f . Phát biểu nào sau đây đúng

A. 0a . B. 0a . C. 1a . D. 0a .

Câu 5: Tích hai nghiệm phương trình 2 2020 0x x bằng : A. 4. B. 1. C. không tồn tại. D.0.

Câu 6: Cho hình vuông cạnh bằng 2 2 . Đường chéo hình vuông bằng

A.8 B.4 C. 2 3 D.3 2

Page 66: SỞ GIÁO DỤC VÀ ĐÀO TẠO NAM ĐỊNH

66

Câu 7: Một khối trụ có đường cao bằng 3 và bán kính đáy bằng 2, xét hai điểm M và N thuộc khối trụ. Khoảng cách lớn nhất giữa hai điểm MN bằng A.2 B.4 C.4 D.5 Câu 8: Xét khối trụ và khối nón có bán kính đáy và thể tích bằng nhau. Tỉ số giữa đường cao khối trụ và khối nón bằng

A.3 B.1

3 C.1 D.

1

2

PHẦN II: Tự luận ( 8 điểm )

Câu 1: ( 1,5 điểm ) Cho biểu thức 2

3 6 4

1 1 1

x xP

x x x

1) Tìm điều kiện xác định của biểu thức P. 2) Rút gọn P và tìm x thoả mãn P = 2.

Câu 2: ( 1,5 điểm ) Cho phương trình: 2 2 2 1 0x mx m ( m là tham số ) (1)

1) Giải phương trình (1) với m = 2.

2) Tìm m để phương trình (1) có hai nghiệm 1 2,x x sao cho:

2

1 1

2

2 2

2 3 2

25 2 2

x mx

x mx

Câu 3: ( 1,0 điểm ) Quãng đường AB dài 50 km. Hai xe máy khởi hành cùng một lúc từ A đến B.Vận tốc xe thứ nhất lớn hơn vận tốc xe thứ hai 10 km/h nên xe thứ nhất đến B trước xe thứ hai 15 phút. Tính vận tốc của mỗi xe. Câu 4 (3,0 điểm): Cho đường tròn tâm (O), từ điểm M ở bên ngoài đường tròn (O) kẻ các tiếp tuyến MA, MB (A, B là các tiếp điểm), kẻ cát tuyến MCD không đi qua tâm O (C nằm giữa M và D; O và B nằm về hai phía so với cát tuyến MCD). a) Chứng minh: tứ giác MAOB nội tiếp.

b) Chứng minh: 2 .MB MC MD

c) Gọi H là giao điểm của AB và OM. Chứng minh: AB là phân giác của CHD

Câu 5 (1,0 điểm): Giải phương trình 3 22 3 4 1x x x x x

HƯỚNG DẪN GIẢI - ĐÁP ÁN

ĐỀ SỐ 01

Bài 1: (2,0 điểm)

Đáp án Điểm

1) Biểu thức 1

1x xác định

1 0

10

1

x

x

1 0x 1.x

0,5

2) Đồ thị hàm số 2y x cắt trục Oy tại điểm M, suy ra tọa độ của điểm M là nghiệm của

hệ phương trình 2

0

y x

y

2

0

x

y

. Vậy 2;0M .

0,5

3) 2105 50

2R S R .

0,5

4) Do 1 2 0 nên hàm số đã cho nghịch biến 0,5

Page 67: SỞ GIÁO DỤC VÀ ĐÀO TẠO NAM ĐỊNH

67

5) Điều kiện để 2 đường thẳng vuông góc là 22 1 .1 1 0m m

Bài 2: (2,5 điểm)

Đáp án Điểm

Phương trình 1 có 2 nghiệm phân biệt 2 20 4 4 1 0m m m 0,25

4 1 0 1.m m 0,25

Với 1m , khi đó phương trình 1 có hai nghiệm phân biệt 1 2,x x .

Theo định lý Viet ta có 1 2

21 2

2

. 1

x x m

x x m m

0,25

Do đó 2 2 2 21 2 1 2 1 2 1 2 1 21 1 2 1 2x x x x x x x x x x 0,25

2

1 2 1 2 1 2 1x x x x x x 0,25

2 2 2

0

4 1 2 1 3 0 1

3

m

m m m m m mm

(thỏa mãn 1m ).

Vậy 1

0;3

m m là các giá trị cần tìm.

0,25

Hệ đã cho tương đương với 2 2 2 2

2 2 2 2

1 2 2 2 2

2 2 2 3 4 2 2 2 3 4

x y y x y y

x y x y x y x y

0,25

22 2 22

2 22 2 2 2

1 5 10 5 02 2 2 2 1

y xx y y x

x y y x xx x x

0,25

2

2 2 2 2 1

1 05 1 0

y x y x x

x yx

. 0,25

Vậy hệ phương trình đã cho có nghiệm là ; 1;0 .x y 0,25

Bài 3: (3,0 điểm)

Đáp án Điểm

Vì ||DN AB AN DB ICA DCB

0,5

Xét ,IAC BDC có ICA DCB và

IAC CDB

IC BCIAC BDC

IA BD ∽ 0,5

2) (1,0 điểm) Xét ,IBC ADC có IDC ADC và

0,25

d

K

H

I

C

N

B

M

O

A

D

Page 68: SỞ GIÁO DỤC VÀ ĐÀO TẠO NAM ĐỊNH

68

1 1 1 1

2 2 2 2ICB sd NB sd ND sd DB sd ND sd NA sd DA ACD

IC ACIBC ADC

IB AD ∽ (1)

Mặt khác ta có MB là tiếp tuyến của đường tròn O nên MBC MDB

MC BCMBC MDB

MB BD ∽ (2)

0,25

Chứng minh tương tự ta có MC AC

MAC MDAMA AD

∽ (3)

Mà MA MB nên MC MC

MB MA .

0,25

Kết hợp với (2) và (3) suy ra AC BC

AD BD (4). Từ (1), (4) và câu 1) suy ra

IC ICIA IB

IB IA 0,25

3) (1,0 điểm)

Vì IA IB nên I là trung điểm của AB , do đó , ,O I M thẳng hàng và OM AB tại I .

Kẻ OH d tại H , OH cắt đường thẳng d tại K . Khi đó ta có 2

2. .OI OK OB

OIK OHM OK OH OI OM OB OKOH OM OH

0,5

Mặt khác vì OB không đổi , ,d O cố đinh nên OH không đổi nên OK không đổi

nên suy ra K cố định. Mà 090OIK nên I thuộc đường tròn đường kính OK cố định 0,5

Bài 4: (1 điểm)

Đáp án Điểm

Điều kiện: 2 5.x Khi đó phương trình 3 28 19 12 3 5 2 2 2x x x x x x x x

23 5 4 3 5 3 2x x x x x x x

23 5 4 5 2 0x x x x x 2

3

5 4 5 2 1

x

x x x x

.

0,25

Giải phương trình (1).

+) Với 2 4x phương trình (1) vô nghiệm vì 2 5 4 0

5 2 0

x x

x x

. 0,25

+) Với 4 5x phương trình (1) 23 2 4 5 7 11 0x x x x x x . 0,25

Ta có 3 2 0, 4;5x x x và 4 5 0, 4;5x x x do đó phương trình

2 22 2

23 2 4 5

7 11 03 2 4 5

x x x xx x

x x x x

2 1 17 11 1 0

3 2 4 5x x

x x x x

2

7 5

27 11 07 5

2

x TM

x x

x Ko TM

(vì 1 1

1 0, 4;53 2 4 5

xx x x x

).

0,25

Page 69: SỞ GIÁO DỤC VÀ ĐÀO TẠO NAM ĐỊNH

69

Vậy phương trình đã cho có tập nghiệm 7 5

3; .2

S

Bài 5: (1,0 điểm)

Đáp án Điểm

(1,0 điểm) Với , , 0a b c ta có: 3 3 23 3

2 .16 16 23 3

a b bc a b bca a a

b bc b bc

Vậy

3 23

116 23

a b bca a

b bc

0,25

Tương tự ta có:

3 23

216 23

b c cab b

c ca;

3 233

16 23

c a cac c

a ca

Ta lại có: 2 2 21 14

16 16 a b c ab bc ca

0,25

Ta có: 2

2 2 2 a bc b ac c ab a b c bc ca ab

2 22 2 2 a bc b ac c ab a b c 2 2 23 3

516 16

a b c a bc b ac c ab

Công theo vế các bất đẳng thức (1), (2), (3),(4) và (5) ta có:

3 3 3 2 2 2

643 3 3

a b c a b c

b bc c ca a ca

0,25

Từ giả thiết ta có 2 22 2 2 212 32 2.2 2a b c b c a b c b c

22 2 2 2 2 2 28 2.2 2 4 2 2 a b c b c a b c b c

2 2 2 2 2 212 4 3 7 a b c a b c

Từ (6) và (7) suy ra 3 3 3 3

43 3 3

a b c

b bc c ca a ca

0,25

ĐỀ SỐ 02

Phần I : Trắc nghiệm: (2 điểm) Mỗi câu trả lời đúng được 0,25 điểm.

Câu 1 2 3 4 5 6 7 8

Đáp án B C D A B B C A

Phần II : Tự luận

Câu Đáp án Điểm

Câu 1

Page 70: SỞ GIÁO DỤC VÀ ĐÀO TẠO NAM ĐỊNH

70

1.a)

a) 3 13 6

2 3 4 3 3

= 3 2 3 13 4 3

2 34 3 16 3

0,25

= 6 3 3 4 3 2 3 0,25

= 10 0,25

1.b)

b) x y y x x y

xy x y

với x > 0 ; y > 0 ; x y

= xy x y x y x y

xy x y

0,25

= x y x y 0,25

= 2 x 0,25

Câu 2

2.a)

Hoành độ giao điểm của đường thẳng (d) và parabol (P) là nghiệm của phương

trình:

x2 = (k 1)x + 3 x2 (k 1)x 3 = 0 (1) 0,25

Phương trình trên là phương trình bậc hai có các hệ số a và c trái dấu nên phương

trình có 2 nghiệm phân biệt với mọi giá trị của k.

Vậy đường thẳng (d) và parabol (P) luôn cắt nhau tại 2 điểm phân biệt 0,25

2.b)

Gọi (x1; y1) và (x2; y2) là tọa độ các giao điểm của (d) và (P). Ta có x1, x2 là các

nghiệm của phương trình (1), áp dụng hệ thức Vi-et, ta có: 1 2

1 2

x x k 1

x x 3

0,25

Có: 2 21 1 2 2y x ; y x . Khi đó điều kiện y1 + y2 = y1 y2 trở thành: 2 2 2 2

1 2 1 2x x x x 0,25

(x1 + x2)2 2x1x2 = (x1 x2)

2. 0,25

Do đó: (k 1)2 + 6 = 9 (k 1)2 = 3 k 3 1

Vậy k 1 3 hoặc k 1 3 thoả mãn đầu bài. 0,25

Câu 3

2 1 2 1 2 1 2 1 2 1 2 1 2 1

2 2 1 2 2 1

x y x y

x y x y

0,25

2 1 1

2 2 1

x y

x y

2 1 1

2 1 2 2

x y

y

0,25

2 1 1 2 1

22

x y x

yy

0,25

KL: Vậy hệ có nghiệm là 2 1

2

x

y

0,25

Page 71: SỞ GIÁO DỤC VÀ ĐÀO TẠO NAM ĐỊNH

71

Câu 4

4.a)

Ta có 090APB (góc nội tiếp chắn nửa đường tròn) 090BPM (kề bù với góc

APB ) 0,25

Lại có 090BCM (...) 0,25

0180BPM BCM 0,25

Do đó tứ giác BPMC nội tiếp. Suy ra 4 điểm B, P, M, C cùng thuộc một đường

tròn. 0,25

4.b)

Chứng minh được:

+) 090OPA KPM (1) 0,25

+) 090OAP KMP (2) 0,25

+) OPA OAP (3) 0,25

Suy ra KPM KMP do đó tam giác KPM cân tại K. 0,25

4.c)

Có: BP AM (vì 090APB ) nên BP là đường cao của tam giác ABM

MC AB nên MC là đường cao của tam giác ABM

AQ BM (vì 090AQB ) nên AQ cũng là đường cao của tam giác ABM

Do đó BP, MC và AQ đồng quy tại một điểm. Gọi điểm đồng quy đó là N.

0,25

Có MK = KP (tam giác KPM cân tại K) (4)

KPM KMP (chứng minh trên) KPN KNP (phụ với hai góc bằng nhau)

KPN cân, do đó KP = KN (5)

0,5

A O I

P

M

d

C

K

B

Q N

Page 72: SỞ GIÁO DỤC VÀ ĐÀO TẠO NAM ĐỊNH

72

Từ (4) và (5) suy ra KM = KN

Chứng minh được KP = KQ ( cùng bằng một nửa của MN)

Chứng minh được hai tam giác KQO và KPO bằng nhau(c-c-c)

Suy ra 090OQK KQ OQ tại Q

Suy ra KQ là tiếp tuyến của đường tròn (O).

0,5

Câu 5 Giải phương trình 2 22 3 2 2 2x x x x x .

Có 22 2 3 1 2 2x x x và

22 1 72 1 7

4 4x x x

với mọi giá trị

của x nên phương trình trên xác định với mọi giá trị x thuộc R.

Đặt a = 2 2 3x x và b = 2 2x x (7

2;2

a b )

Ta có 2 2

2 2

1

a b x

a b x

0,25

2 22 2 2 1 0a b a b a b a b

2 2 1 0a b (vì a, b > 0) 2 2 1a b 0,25

2 2 2 2 2

2

2 2

2 2 3 2 2 1 4 2 3 4 2 4 2 1

32344 3 4 28

4 3 16 2

x x x x x x x x x x

xx x x x

x x x

Thay x = 23

8 vào pt đã cho thấy thỏa mãn.

KL: Vậy phương trình có nghiệm 23

8x

0,5

ĐỀ SỐ 03

Câu 1 (2,0 điểm)

Đáp án Điểm

1) Biểu thức 1

1x xác định

1 0

10

1

x

x

1 0 x 1. x

0,5

2) Đồ thị hàm số 2 2 y x cắt trục Oy tại điểm M, suy ra tọa độ của điểm M là nghiệm của

hệ phương trình 2 2

0

y x

x

0

2

x

y. Vậy 0;2M .

0,5

3) Hình vuông ABCD nội tiếp trong đường tròn ( ; )O R suy ra O là tâm của hình vuông và

2 2 2 21 1 1 55 5

2 2 2 2 R OA AC AB BC .

0,5

4) Khi 3x ta có 3. 1 2 3 5 4 3 3. 1 5 2 3 4 3 6 P .Vậy 6P . 0,5

Page 73: SỞ GIÁO DỤC VÀ ĐÀO TẠO NAM ĐỊNH

73

Câu 2 (2,0 điểm)

Câu 3 (2,5 điểm)

Đáp án Điểm 1) (1,5 điểm)

a) (0,5 điểm) Phương trình * có nghiệm 20 4 4 1 0 m 0,25

12 4 0 3 m m . 0,25 b) (1,0 điểm)

Với 3 m phương trình * có hai nghiệm 1 2;x x . Theo định lý Viet ta có 1 2

1 2

4

. 1

x x

x x m 0,5

Do đó: 3 31 2 1 21 1 28 x x x x

3

1 2 1 2 1 2 1 2 1 23 . 1 . 28 x x x x x x x x x x 0,25

64 12 1 1 4 1 28 m m 11 22 2 m m (Thỏa mãn điều kiện 3 m ).

Vậy 2. m 0,25

Điều kiện:

2

2

1 04

4 01 3 0 lu«n ®óng y

2 4 0

xx

xy

y y

Hệ phương trình đã cho

2

2

1 4 2 4 7

1 4 2 4 1

x x y y y

x x y y y

0,25

Đáp án Điểm 1) (1,0 điểm)

Với điều kiện 0x và 1x , ta có

2 2 2 2

: :11 1 1 11 1

x x x xP

xx x x xx x

0,25

2 1 2 1.

1 1

x x x

xx x

3 1.

1 1

x x x

xx x

0,5

3 1 3.

1 1

x x x

x x x

0,25

2) (0,5 điểm) Với 0x và 1x , ta có 3

1

xP

x

5 3 52 6 5 5

2 21

xP x x

x

0,25

1 13 1

3 9x x x .( thỏa mãn điều kiện)

Vậy với 1

9x thì

5

2P .

0,25

Page 74: SỞ GIÁO DỤC VÀ ĐÀO TẠO NAM ĐỊNH

74

2

2

1 4 1 3 1 6

1 4 1 3 1 2

x x y y

x x y y

2

2

1 4 1 3 1 6

3 32

1 4 1 3 1

x x y y

x x y y

Vì 1 4 0 4x x x ; 2 2 2

1 3 1 1 1 1 3 1 0y y y y y y y

Đặt

2

1 4

1 3 1

a x x

b y y

( ; 0a b )

Hệ phương trình trở thành

6 6 6 33 3

3 2 9 32

a b a b a b a

a b ab ab ba b

0,25

+)

2

1 4 3 13

3 1 3 1 3 2

x xa

b y y

Giải phương trình (1), ta có:

1 4 3x x 1 4 2 1 4 9x x x x 1 4 7x x x

2

7 0 75

51 4 7

x xx

xx x x

0,25

Giải phương trình (2), ta có: 2

1 3 1 3 y y 2

1 3 3 1 y y

22

3 1 0 42

21 3 3 1

y yy

yy y. Vậy HPT đã cho có nghiêm ; 5;2x y .

0,25

ĐỀ SỐ 04

I. Trắc nghiệm Mỗi ý đúng cho 0,25 điểm Câu 1 2 3 4 5 6 7 8

Đáp án C A D B D A B B

II. Tự luận (8 điểm)

Câu 1.(1,5 điểm) Cho biểu thức A = x 1 2 3

+ x + 2 x + 1 x + 3 x + 2

( với x 0 ).

1) Rút gọn biểu thức A.

2) Chứng minh rằng khi x = 4 2 3 thì biểu thức A có giá trị bằng 3 1 .

Đáp án Điểm

1) 1,0 điểm +) Với x 0 ta có:

3 2 2 2x x x x x

1 2 1

1 2

x x x

x x

0,25

+)

1 2 3A

2 1 2 1

x

x x x x

1 1 2 2 3

2 1

x x x

x x

0,25

Page 75: SỞ GIÁO DỤC VÀ ĐÀO TẠO NAM ĐỊNH

75

1 2 4 3 2

2 1 2 1

x x x x

x x x x

0,25

2

.12 1

x x x

xx x

0,25

2) 0,5 điểm

Với x = 4 2 3 , ta có:

A 4 2 3

4 2 3 1

0,25

2

2

3 1 3 1 2 33 1 3 13 1

3 1 1 3 2 2 3 2 33 1 1

.

Vậy với x = 4 2 3 thì A nhận giá trị bằng 3 1 .

0,25

Câu 2. (1,5 điểm) Cho phương trình x2 – 3(m – 1)x + 2m2 – 6m = 0 (1)

1) Chứng minh phương trình (1) luôn có nghiệm với mọi giá trị của m. 2) Gọi x1, x2 là các nghiệm của phương trình (1). Chứng minh giá trị của biểu thức:

B = 2

1 2 1 2 1 22 6 6 9x x x x x x không phụ thuộc vào giá trị của m.

Đáp án Điểm 1) 0,5 điểm

Phương trình (1) có = 9(m – 1)2 – 4(2m2- 6m) 0,25 = 9m2 – 18m + 9 – 8 m2 + 24 m = m2 + 6m + 9 = (m +3)2 0 với mọi giá trị của m Do đó phương trình (1) có nghiệm với mọi giá trị của m.

0,25

2) 1,0 điểm Áp dụng hệ thức Vi-ét ta có: x1 + x2 = 3(m – 1) x1.x2 = 2m2 – 6m.

0,25

Ta có B = 2

1 2 1 2 1 22 6 6 9x x x x x x = 2

1 2 1 2 1 22 6 9x x x x x x

= 2.9(m – 1)2 – 6.3(m – 1) – 9(2m2 – 6m) 0,5

B = 18m2 – 36m + 18 – 18 m + 18 – 18 m2 + 54 m = 36.

0,25

Câu 3. (1,0 điểm) 1) Chứng minh rằng hệ phương trình luôn có nghiệm duy nhất . 2) Giả sử (x0 ; y0) là nghiệm của hệ phương trình trên. Khi m thay đổi, chứng minh rằng điểm

M(x0; y0) luôn thuộc một đường thẳng cố định. Đáp án Điểm

1) Giải hệ phương trình:

2

2

( 1) 1

2 2

mx y m

x my m

Từ (2) suy ra 2x 2my m , thay vào phương trình (1) ta được:

2 2 2 3 2

2 2

2 2 1 1 2 2 1

1 1 2 1 2 1

m my m y m m m y m m m

m y m m y m

0,5

Page 76: SỞ GIÁO DỤC VÀ ĐÀO TẠO NAM ĐỊNH

76

Với 2 1y m suy ra x = m .

KL... 0,25

2) (x0 ; y0) là nghiệm của hệ phương trình trên. Do đó ta có

0 02 1; .y m x m Do đó 0 02x 1.y

Suy ra điểm M(x0; y0) luôn thuộc đường thẳng cố định là 2x 1.y 0,25

Câu 4 : ( 3 điểm ) Cho đường tròn (O; R) và điểm A nằm ngoài đường tròn (O). Qua A vẽ tiếp tuyến AB tiếp xúc với đường tròn (O) tại B. Vẽ một đường thẳng qua A cắt đường tròn tại hai điểm M và N (M nằm giữa A và N). Qua M kẻ đường thẳng song song với AB cắt BN tại E. Gọi I là trung điểm của ME. Vẽ dây BQ của đường tròn (O) sao cho BQ đi qua điểm I. a) Chứng minh hai tam giác BMI và BQM đồng dạng. b) Chứng minh tứ giác QIEN nội tiếp. c) Chứng minh BM. QN = BN. MQ.

Đáp án Điểm a) 1,0 đ

Có BMI MBA ( so le trong)

Mà MBA MQB (góc nội tiếp, góc tạo bởi tia tiếp tuyến và dây cung cùng chắn một cung)

MQB BMI

0,5

Xét tam giác BMI và tam giác BQM có MQB BMI và B chung

Vậy BMI BQM (g – g) 0,5

b) 1,0 đ

Có BIE ABQ (cặp góc so le trong) 0,25

Mà ABQ ENQ (góc nội tiếp, góc tạo bởi tia tiếp tuyến và dây cung cùng chắn một cung) 0,25

Do đó BIE ENQ 0,25

Do đó tứ giác QIEN nội tiếp (t/g có góc ngoài tại một đỉnh bằng góc trong ở đỉnh đối diện). 0,25 c) 1,0 đ

Ta có BMI BQM (CM trên) MI MB

IB MQ (1) 0,25

Ta có BIE ENQ nên chứng minh được BIE BNQ EI BN

IB NQ (2) 0,5

Từ (1) và (2) ta có MB BN

MQ NQ , từ đó ta có ĐPCM. 0,25

Câu 5. (0,5 điểm) Giải phương trình 2 2 2x x 1 x x 1 x x 2 . Đáp án Điểm

A

B

N E

O

M

Q

I

Page 77: SỞ GIÁO DỤC VÀ ĐÀO TẠO NAM ĐỊNH

77

Điều kiện xác định: 2

2

x x 1 0

x x 1 0

(*)

0,25 Với ĐK (*) ta có :

2 21 x x 1 2 x x 1 (1)

và 2 21 x x 1 2 x x 1 (2)

Cộng vế tương ứng của (1) và (2), ta có

2 2 2 22x 2 2 x x 1 x x 1 x 1 x x 1 x x 1 (3)

0,25 Nếu phương trình đã cho có nghiệm

2 2 2x x 1 x x 1 x x 2 (4)

Từ (3) và (4) 2 2x x 2 x 1 (x 1) 0 x 1 (TMĐK*)

Thử lại x = 1 vào phương trình đã cho, ta thấy nghiệm đúng . Vậy nghiệm của phương trình là x = 1.

ĐỀ SỐ 05

Câu Nội dung Điểm Câu 1 1) Tìm tập nghiệm của phương trình 2 3 .x x

2) Tìm m để đường thẳng 2 1y x và đường thẳng 2 3y x m cắt nhau tại

một điểm trên trục tung.

3) Lập một phương trình bậc hai ẩn x nhận 2 3 và 2 3 là nghiệm.

4) Cho ( ; )O R và dây 2AB R . Tìm số đo cung lớn .AB

2,0

1)

2

02 3 3.

2 3

xx x x

x x

Tập nghiệm của phương trình đã cho là 3S .

0,5

2) Vì đường thẳng 2 1y x và 2 3y x m có hệ số góc khác nhau nên chúng cắt

nhau tại một điểm trên trục tung 1 3 4m m

0,5

3) Ta có 2 3 2 3 4; 2 3 2 3 1.

Phương trình bậc hai ẩn x nhận 2 3 và 2 3 là nghiệm cần lập là 2 4 1 0x x

0,5

4) Vì dây 2AB R nên AB là độ dài cạnh một hình vuông nội tiếp (O;R). Số đo cung lớn AB bằng 2700.

0,5

Câu 2 Cho biểu thức

2 21

1

x x x xA

x x x

(với 0x ).

a) Rút gọn .A

b) Chứng minh rằng nếu 0 1x thì 0.A A

1,5

1) Với 0x , ta có

1 2 11

1

1 1 2 1

.

x x x x xA

x x x

A x x x

A x x

0,5 0,5

Page 78: SỞ GIÁO DỤC VÀ ĐÀO TẠO NAM ĐỊNH

78

2) Do 0 1x nên 0x x x x

0 0.A A A A A

0,25 0,25

Câu 3 1) Cho phương trình 2 3( 1) 3 4 0x m x m (với x là ẩn, m là tham số).

a) Chứng minh rằng với mọi giá trị của m thì phương trình đã cho luôn có nghiệm. b) Tìm các giá trị của tham số m để phương trình có hai nghiệm phân biệt là

độ dài hai đường chéo của một hình thoi có độ dài cạnh là 5

.2

2) Cho hệ phương trình 2

3

2

x my m

mx y m

(với ,x y là ẩn, m là tham số). Tìm m

để hệ phương trình có nghiệm duy nhất ;x y thỏa mãn 2 2 0.x mx y

2,5

1.a) Ta có 1 3 3 3 4 0a b c m m nên với mọi m thì phương trình đã cho luôn có hai nghiệm là 1 và 3 4.m

0,5

1.b) Phương trình có hai nghiệm phân biệt

51 3 4 .

3m m

Theo bài ra thì hai nghiệm của phương trình phải dương 4

3 4 03

m m ( vì

1>0). Hai nghiệm của phương trình là độ dài hai đường chéo của một hình thoi có độ dài

cạnh là 22 2

4

35

2 1 3 4 5

2 2 2

m

m

22

4 43 23

3 4 21 3 4 5

m mm

mm

(thỏa mãn)

Vậy 2.m

0,25 0,25 0,25 0,25

2)

2

3 (1)

2 (2)

x my m

mx y m

Từ (1) ta có 3x m my , thay vào (2) ta có 2; .y x m Suy ra với mọi m hệ luôn có nghiệm duy nhất .

Khi đó 2 2 2 22 0 2 2 0 2 2 2.x mx y m m m m

0,5 0,5

Câu 4 Cho tam giác ABC có ba góc nhọn, nội tiếp đường tròn (O) (AB < AC). Các tiếp tuyến với (O) tại B và C cắt nhau tại N. Vẽ dây AM song song với BC. Đường thẳng MN cắt đường tròn (O) tại M và P.

1) Chứng minh rằng ON vuông góc với BC. Tính độ dài đoạn BC (đơn vị cm)

biết 2 2

1 1 1.

25OB NC

2) Chứng minh rằng .BP CP

AC AB

3) Chứng minh rằng BC, ON và AP đồng quy.

3,0

Page 79: SỞ GIÁO DỤC VÀ ĐÀO TẠO NAM ĐỊNH

79

1) - Chỉ ra ON là đường trung trực của BC .ON BC Gọi K là giao điểm của ON và BC thì K là trung điểm của BC. - Ta có OBN vuông tại B, BK là đường cao nên

2 2 2 2 2

1 1 1 1 1.

OB NC OB NB BK

Kết hợp giả thiết suy ra 2 25 5 10 .BK BK cm BC cm

0,5 0,25 0,25

2) - Chỉ ra NBP và NMB đồng dạng

PB NB

MB NM (1).

Tương tự, NCP và NMC đồng dạng PC NC

MC NM (2).

- Vì NC NB (3) nên từ (1), (2) và (3) suy ra PB PC

MB MC (4).

- Mặt khác, Tứ giác AMCB nội tiếp (O) và / /AM BC Tứ giác AMCB là hình

thang cân. ,MC AB MB AC (5). Từ (4), (5) .PB PC

AC AB

0,25 0,25 0,25 0,25

3) Gọi Q là giao điểm của AP và BC.

- Chỉ ra BQP và AQC đồng dạng BQ PB

AQ AC (6).

Tương tự, CQP và AQB đồng dạng CQ PC

AQ AB (7).

- Kết hợp (6), (7) và kết quả câu b) ta suy ra BQ CQ

BQ CQ QAQ AQ

là trung

điểm của BC. Suy ra Q K . Vậy , ,BC ON AP đồng quy tại K.

0,25 0,25 0,25 0,25

Câu 5 1) Giải phương trình 2 2 5 3 2 1.x x x x

2) Cho a, b là hai số dương thỏa mãn 1a b . Tìm giá trị nhỏ nhất của biểu thức

2 23 3 2 2 3

.2

M a b a b ab

1,0

P

K≡QO

M

N

C

B

A

Page 80: SỞ GIÁO DỤC VÀ ĐÀO TẠO NAM ĐỊNH

80

1) ĐKXĐ: 1.x

2 2

2

2 5 3 2 1 2 1 1 3 3 1 4 0

1 3 1 4 0.

x x x x x x x x x x

x x x x

Đặt 1y x x (điều kiện 1y ), phương trình trở thành

2 3 4 0 1 4 0 4y y y y y (do 1y ).

Ta có phương trình

2

1 4 9 131 4 1 4 .

21 16 8

xx x x x x

x x x

Vậy phương trình có nghiệm 9 13

.2

x

0,25 0,25

2) - Chứng minh được với a, b là hai số dương ta có: 3 3 ( ).a b ab a b

Suy ra 2 23 3 ( )a b ab a b mà 1a b

23 3 2( ) .a b ab

Ta có: 22 2 2 1 2 .a b a b ab ab

- Do đó 2

2 3 1 15 15( ) 1 2 .

2 4 16 16M ab ab ab ab

Dấu “=” xảy ra

11

42

1

aba b

a b

(thỏa mãn).

Vậy giá trị nhỏ nhất của M là 15

16 khi

1.

2a b

0,25 0,25

Page 81: SỞ GIÁO DỤC VÀ ĐÀO TẠO NAM ĐỊNH

81

Câu 4 (3,0 điểm) Đáp án Điểm

1) (1,0 điểm)

Dây BC không đi qua tâm O và H là trung điểm BC 090OK BC DHK (1)

NK là đường kính của (O ; R) 090NAK (2)

0,5

Từ (1) và (2) 4 điểm A, K, H, D thuộc đường tròn đường kính DK Tứ giác AKHD là tứ

giác nội tiếp.

0,5

2) (1,0 điểm) H là trung điểm của BC N là điểm chính giữa cung BNC

sđBN sđCN Các góc NAB và NBD là góc nội tiếp lần lượt chắn cung NB và NC của

(O ; R) NAB NBD

0,5

Góc BDN có đỉnh nằm trong (O ; R) 1

2BDN ( sđBN sđAKC )=

1

2sđACN 0,25

mà góc ABN nội tiếp chắn cung ACN

ABN = 1

2sđACN BDN ABN

0,25

3) (1,0 điểm) MA là tiếp tuyến của (O ; R) 1

2MAN sđ

1

2ABN (sđAB sđBN ) (2)

và 1

MDA2

(sđAB sđCN ) (3).

Mà sđBN sđCN nên từ (2) và (3) MAN MDA MAD cân ở M MA MD .

0,5

Do đường tròn (O) cố định và điểm M cố định, nên MA không đổi MD không đổi.

Vậy D thuộc đường tròn ;M MA cố định . 0,5

Câu 5 (1,0 điểm)

DH

N

K

B

O

A

M

C

Page 82: SỞ GIÁO DỤC VÀ ĐÀO TẠO NAM ĐỊNH

82

Đáp án Điểm

Với các số thực dương , , ,a b x y , ta có bất đẳng thức

22 2 a ba b

x y x y

Thật vậy,

22 2 2 2

2a ba b a bx y a b

x y x y x y

(luôn đúng)

0,25

Từ giả thiết 31 1z z

Ta có

23 2 2 32 2 3

1 1 3 1 3 3 2 3 3

x yx y z x y zP

y x xy xy x xy y xy x y xy xy

0,25

22

12 4

x yx yxy

nên

2

2 2 2

21 4

2 41

2 4

x y x yP

x yx y x y x yx y

0,25

Đặt 2 2t x y t xy . Ta có 2

2 4

2 4

tP

t t

. Ta chứng minh

2

2 4 3

2 4 2

t

t t

Thật vậy, 2 2

2

2 4 34 4 8 2 3 2 4

2 4 2

tt t t t t

t t

33 26 12 8 0 2 0t t t t (luôn đúng với mọi 2t ). Vậy BĐT được chứng minh.

0,25

ĐỀ SỐ 06

Phần I: Trắc nghiệm (2,0 điểm) Mỗi câu đúng cho 0,25 điểm Câu 1 2 3 4 5 6 7 8 Đáp án C B C C B A A D

Phần II: Tự luận (8,0 điểm)

Bài 1. (1,5 điểm) Rút gọn các biểu thức sau:

a) 3 13 48A

3 13 48A 3 13 4 3 0,25

2

3 2 3 1 3 2 3 1 0,25

2

3 1 3 1 0,25

b) 2

:a a b b b

B ab a ba b a b

với 0 0,a , b a b

với 0 0,a , b a b , ta có 0,25

Page 83: SỞ GIÁO DỤC VÀ ĐÀO TẠO NAM ĐỊNH

83

3 3

2 2: :

2 2: 2 :

a ba a b b b bB ab a b ab a b

a b a b a b a b

a b a ab b b bab a b a ab b a b

a b a b a b

2 2 2

:b a b b

a b a ba b a b a b

0,25

2

1a b b a b

a b a b

0,25

Bài 2. (1,5 điểm) Cho hai hàm số 2y x và 3y kx với k là tham số.

a) Khi 2k , tìm tọa độ giao điểm của hai đồ thị hai hàm số trên.

Hoành độ giao điểm của đồ thị hai hàm số 2y x và 3y kx là nghiệm của phương trình 2 3x kx hay 2 3 0x kx (1)

0,25

Với 2k , phương trình (1) có dạng 2 2 3 0x x 0,25

Giải (1) tìm được 1 1x và 2 3x , từ đó suy ra tọa độ giao điểm ( 1;1)P và (3;9)Q 0,25

b) Chứng minh rằng với mọi giá trị k, đồ thị của hai hàm số đã cho luôn cắt nhau tại hai

điểm phân biệt 1 1;A x y và 2 2;B x y . Tìm tất cả các giá trị của m sao cho 2 21 2 36y y .

Xét phương trình 2 3 0x kx (1)

Ta có 2 12 0k với mọi k suy ra (1) luôn có 2 nghiệm phân biệt, hay đồ thị của hai hàm

số đã cho luôn cắt nhau tại hai điểm phân biệt 1 1;A x y và 2 2;B x y

0,25

Theo định lý Vi-ét ta có 1 2x x k , 1 2. 3x x

Lại có 2 21 1y x , 2 2

2 2y x 0,25

22 2 2 2

1 2 1 2 1 2 1 236 35 2 36y y x x x x x x

hay 2 22( 3) 36 30 30k k k

Kết luận: Vậy 30k thỏa mãn đề bài 0,25

Bài 3. (1,0 điểm) Giải hệ phương trình 2 2

1

7

x y

x xy y

1 1x y y x 0,25

Thay 1y x vào phương trình 2 2 7x xy y

ta có 2 2 2( 1) ( 1) 7 6 0x x x x x x 0,25

Giải phương trình trên tìm được 1 2x ; 2 3x 0,25

suy ra 1 3y , 2 2y

Kết luận nghiệm của hệ phương trình đã cho là ( 2; 3); 3; ( 2) 0,25

Bài 4. (3,0 điểm) Cho đường tròn tâm O và dây cung AB. Lấy điểm E trên dây cung AB (E khác A và B). Qua E vẽ dây cung CD của đường tròn (O). Trên hai tia DA và DB lấy điểm P và Q đối xứng qua E. Gọi I là tâm đường tròn đi qua C và tiếp xúc với PQ tại E, đường tròn (I) cắt AB tại M. Chứng minh rằng: a) . .EA EB EC ED b) CMA ∽ QED

c) MA MB .

Page 84: SỞ GIÁO DỤC VÀ ĐÀO TẠO NAM ĐỊNH

84

a) Chứng minh EA.EB = EC.ED

Chứng minh được EAD ∽ ECB 0,25

Suy ra được EA ED

EC EB 0,25

Từ đó suy ra EA.EB = EC.ED 0,25 b) CMA ∽ QED

Chứng minh được CMA PEC (góc nội tiếp và góc tạo bởi tia tiếp tuyến và dây cùng chắn cung EC của đường tròn (I))

0,25

Suy ra được CMA DEQ (hai góc đối đỉnh) (1) 0,25

Chứng minh được CAE CDB (hai góc nội tiếp cùng chắn cung BC của (O)) (2) 0,25

Từ (1) và (2) suy ra CMA ∽ QED 0,25

c) MA MB .

Theo chứng minh trên ta có CMA DEQ suy ra CMB PED (3) 0,25

Chứng minh được CBA CDA (hai góc nội tiếp cùng chắn cung AC của (O)) (4) 0,25

Từ (3) và (4) suy ra MBC ∽ EDP

Suy ra MB ED

CM EP hay

MB ED

CM EQ (do EQ=QP) (5) 0,25

Theo câu b ta có CMA ∽ QED suy ra MA ED

CM EQ (6) 0,25

Từ (5) và (6) suy ra MA MB

CM CM hay MA = MB 0,25

Bài 5. (1,0 điểm)

a) Tìm giá trị nhỏ nhất của biểu thức 9 6 1 2P x x x x .

với 0x ta có

2 2

9 6 1 2 3 1

3 1 3 1 2

P x x x x x x

x x x x

0,25

M

I

Q

E

O

A

B

D

C

P

Page 85: SỞ GIÁO DỤC VÀ ĐÀO TẠO NAM ĐỊNH

85

Dấu “=” xảy ra khi 1; 3x x . Vậy GTNN của P bằng 2 0,25

b) Lập luận đúng được 0,5 điểm Trong một cuộc thi bóng đá thể thức vòng tròn một lượt tính điểm, đội thắng được 3 điểm, đội hòa được 1 điểm, đội thua 0 điểm, có 4 đội tham gia. Cuối cùng 4 đội nhận được lần lượt lần lượt 5, 1, x, 6 điểm. Tìm x? Theo bài ra ta có bảng sau: Đội Thắng Hòa Thua Tổng điểm

1 X x x 0 5

2 x x x 1

3 x

4 x x 0 0 6

Từ bảng trên suy ra: Đội 1 thắng Đội 4, hòa Đội 2 và Đội 3

Đội 2 thua Đội 3 và Đội 4 Đội 3 thua Đội 4, hòa Đội 1, thắng Đội 3 nên được 4 điểm

Vậy x = 4

ĐỀ SỐ 07

Phần I: Trắc nghiệm (2,0 điểm) Mỗi câu đúng cho 0,25 điểm Câu 1 2 3 4 5 6 7 8 Đáp án D C D A B A B C

Phần II: Tự luận (8,0 điểm) Bài 1: (1,5 điểm)

1) Chứng minh đẳng thức thức: 1 33 2 2019

48 1 675 32 11 3 1 3

2

2 3 11 33 2 2019 1 33 2019 348 1 4 .3 1

2 2 11 3 1 311 3 1 3

0,25

2 3 3 3 1 673 3 1 0,25

675 3 0,25

2) Rút gọn biểu thức, 1 2

11 1

xB

xx x

với 0x và 1x .

Với 0x và 1x , ta có

1 1 2 1 2

1 1 1 1

x x x x x xB

x x x x

0,25

2 1

1 1

x x

x x

0,25

Page 86: SỞ GIÁO DỤC VÀ ĐÀO TẠO NAM ĐỊNH

86

2

1 1

11 1

x x

xx x

0,25

Bài 2: (1,5 điểm) 1) (0,5 điểm)

2

2

2

2 3 0 1

1 11' 3

2 4

x mx m

m m m

0,25

Chứng minh ' 0 m , từ đó suy ra được phương trình (1) luôn có hai nghiệm phân biệt 1x ,

2x với mọi m 0,25

2) (1,0 điểm)

Lý luận và tìm được 2 21 1 1 12 3 0 2 3x mx m x mx m (2) 0,25

Lý luận và viết được 1 2 2x x m (3) 0,25

Từ (2), (3) và 21 22 6x mx , tìm được 24 3 0m m 0,25

Tìm được m = 1 hoặc 3

4m

. Vậy 1m hoặc

3

4m thỏa mãn đề bài 0,25

Bài 3: (1, 0 điểm)

2 16 1

2 2

x xy y

x y

Điều kiện 0x và 0y

Ta có 2

1 16x y (3) 0,25

Từ (3), lập luận và tìm được 4 4x y 0,25

Từ (2) và (4) ta có 4 2 6

22

x y x

y xx y

0,25

3 9

11

x x

yy

(thỏa mãn điều kiện). Vậy hệ đã cho có 1 nghiệm duy nhất ; 9; 1 .x y 0,25

Bài 4. (3,0 điểm)

1) Chứng minh tứ giác ACIH nội tiếp đường tròn và AIH ABD . (1,0 điểm)

Chứng minh 090ACI , 090AHI 0,25

Chứng minh tứ giác ACIH nội tiếp đường tròn 0,25

Chứng minh tứ giác BDIH nội tiếp đường tròn ( 090BDI , 090BHI ) 0,25

Chứng minh AIH ABD (1) 0,25

I

BOH

M

A

C

D

K

1

1

1

Page 87: SỞ GIÁO DỤC VÀ ĐÀO TẠO NAM ĐỊNH

87

2) Chứng minh KCH KOD ∽ . (1,0 điểm)

Chứng minh 1I ACH (2) (tứ giác ACIH nội tiếp đường tròn)

Chứng minh KCA ABD (3) (tứ giác ACDB nội tiếp đường tròn) 0,25

Từ (1), (2) và (3) suy ra được ACK ACH OBD

Từ đó suy ra được 2.KCH OBD (4) 0,25

Chứng minh 1D OBD từ đó suy ra

1 2.O OBD (5)

Từ (4) và (5) suy ra được 1O KCH 0,25

Chứng minh KCH KOD ∽ ( 1,OKB chung O KCH ). 0,25

3) Chứng minh ba điểm I, H, M thẳng hàng Chứng minh . .KC KD KH KO (6) (vì KCH KOD ∽ ) 0,25

Chứng minh KMC KDM ∽ (g-g) từ đó suy ra 2.KC KD KM (7) 0,25

Từ (6) và (7) suy ra được KH KM

KM KO

Chứng minh KHM KMO ∽ ( ,KH KM

OKM chungKM KO

) 0,25

Chứng minh 090KHM KMO suy ra MH AB tại H Kết hợp với IH AB tại H suy ra I, H, M thẳng hàng.

0,25

Bài 5. (1,0 điểm) Chứng minh rằng: 2 22 2

11

xy z x y

xy

Với x, y, z là các số dương và x + y + z = 1, ta có

2

0 2 2 1 2x y x y xy x y z xy z xy z 0,25

2

2 2

(1)

xy z xy z z xy xy z xy z

xy z xy z

0,25

2 2 2 2 2 2 2

2 2

0 2 2 2

2 2

x y x y xy x y x y xy

x y x y

0,25

Từ (1) và (2) ta có 2 22 2 1

11 1 1

xy z x y xy x x y xy

xy xy xy

(đpcm) 0,25

ĐỀ SỐ 08

Câu 1) 2

21

px x

dấu bằng xảy ra khi x=0

Câu 2) HD 22 22 2x x m

Câu 3) đs 54

Câu 4)

Page 88: SỞ GIÁO DỤC VÀ ĐÀO TẠO NAM ĐỊNH

88

HD: 2) ( . . ) ;ACM BCE c g c CM CE ACM BCE MCE ACB

Do 090ACB nên 090MCE ECM vuông cân tại C.

3) Vì d là tiếp tuyến , AB là đường kính (O) suy ra d vuông góc AB tại A

Suy ra PAM ABM OBM BMO lại có từ giả thiết

0

. . .

90

AP MOAP MB MA R MAOM APM MOB

MA MB

AMP MBO ABM OMB PMO AMB

Suy ra PM là tiếp tuyến của đường tròn (O). Gọi Q là giao điểm của tia BM và d

Chứng minh : PA=PQ=PM

Có HK// AQ nên IH IK

IH IKPQ AP

; I là giao điểm BP và HK .

Câu 5) pt 221 1 1x x y y chuyển vế, liên hợp, nhóm nhân tử suy ra

x=-y, (phần còn lại chứng minh vô nghiệm)

Thế x = - y vào (2) 2 22

2

2

2 6 1 3252 6 1

2 4 2 6 1 2

x x xx xx x

x x x

Với 22 6 1 3 1; 1x x x x y

Page 89: SỞ GIÁO DỤC VÀ ĐÀO TẠO NAM ĐỊNH

89

+ 2 3 11 3 112 6 1 2 ;

2 2x x x x y

Kết luận pt có nghiệm 1

1

x

y

hoặc

3 11

2

3 11

2

x

y

ĐỀ SỐ 09

Câu 1. 1) Biểu thức P xác định

01

01

01

2x

x

x

1

1

x

x

3 6 4 ( 1) 3( 1) (6 4)

1 1 ( 1)( 1) ( 1)( 1)

x x x x x xP

x x x x x x

2 2 23 3 6 4 2 1 ( 1) 1( 1)

( 1)( 1) ( 1)( 1) ( 1)( 1) 1

x x x x x x x xvoi x

x x x x x x x

Câu 2)

Phương trình (1) luôn có hai nghiệm x1, x2 với mọi m

2

3

4 2 1 2 50 4 6 54 0 3 2 9 0 9

2

m

m m m m m mm

Câu 3

Gọi vận tốc xe thứ nhất là x km/h ( x >10) thì vận tốc xe thứ hai là x - 10 km/h

Thời gian xe thứ nhất đi từ A đến B là 50

x h

Thời gian xe thứ hai đi từ A đến B là 50

10x h

Theo đề bài ta có phương trình 50 50 1

10 4x x

250 ( )

10 2000 0 ( 50)( 40) 040 ( )

x Nx x x x

x L

2 2' 2 1 ( 1) 0m m m

Vì x1, x2 là là hai nghiệm của phương trình (1) nên ta có:

2

1 1

2

2 2

2 3 4 2

2 2 1 2

x mx m

x mx m

Theo đề bài 2 2

1 1 2 22 3 2 2 50x mx x mx

Page 90: SỞ GIÁO DỤC VÀ ĐÀO TẠO NAM ĐỊNH

90

Câu 4

Câu 5)

3 2 3 22 3 4 1 4 4 2 2 1 3 0x x x x x x x x x x

2 2

2 2 1 02 2 1 3

x xx x x

x x

1 12 2 1 0

2 2 1 3x x x

x x

1 1 1 1

2 2 1 03 32 2 1 3

x x xx x

1 12 2 1 0

3 2 2 2 1 3 1 3 2 3

x xx x x

x x x x

1 12 1 2 0

3 2 2 2 1 3 1 3 2 3x x x

x x x x

2 1 0 2, 1;x x x x

2) ét à ó:X MBC v MDB c

BMD

1( )

2

chung

MBC MDB sd BC

2MBC MDB (g-g) . (1)MB MC

MB MC MDMD MB

0 23) MOBcó 90 ; . (2)B BH OM MB MH MO

(1) & (2) MC.MD = MH.MO

ét MCH & MOD có:

chung

( ì MC.MD = MH.MO)

X

DMO

MC MHv

MO MD

MCH MOD (c.g.c) MHC ODM (3)

tứ giác OHCD nội tiếp ; à ( cân) (4)OHD OCD m OCD ODM OCD OHD ODM

0(3) & (4) 90MHC OHD do MHC CHB OHD DHB

CHB DHB AB là phân giác của CHD